Download as pdf or txt
Download as pdf or txt
You are on page 1of 91

 

 
 

1.1 If p and q are chosen randomly from set right. What is the probability that the
{1, 2, 3, 4, 5, 6, 7, 8, 9, 10} with walker returns to the starting pt after 4
replacement find probability that the steps
roots of x 2  px  q  0 are real 1 1
(A) (B)
p ( p 2  4 q  0) 2 4
3 3
62 60 (C) (D)
(A) (B) 8 4
100 100
1.6 Entries of a 2  2 determinant are
38 74
(C) (D) chosen the set [1, 1] the probability
100 100
1.2 A box contains 2 washes, 3 nuts and 4 that determinant has zero value is
bolts items Are drawn from the box at 1 4
(A) (B)
random one at a time without 2 7
replacement the probability of drawing 2 3 3
(C) (D)
washes, first followed by 3 nuts and 2 8
subsequently 4 bolts is 1.7 Two coins R and S are tossed. The 4
1 63 joint events H R H S , TRTS , H RTS , TR H S
(A) (B)
1260 1260 have probabilities 0.28, 0.18, 0.30, 0.24
62 respectively, where H represents head
(C) (D) Not
1260 and T represents tail. Which one of the
1.3 Four squares are chosen at random on a following is TRUE?
chessboard, find probability that. (A) The coin tosses are independent.
1.4 A fair coin is jessed n times the (B) R is fair, S is not.
probability that the difference between (C) S is fair, R is not.
the no. of heads and tails is ( n  3) is (D) The coin tosses are dependent.
1 1.8 The probability that the system of
(A) 0 (B)
2n equation has a unique solution
1 3 3
(C) (D) Not (A) (B)
2 8 4
1.5 In one dimension a random walker takes 5 1
(C) (D)
a steps with equal problem to the left or 8 2

 
2 | Engineering Mathematics  
1.9 A box contains 4 white balls and 3 red 1 4
(A) (B)
balls. In succession, two balls are 2 9
randomly selected and removed from 5 6
the box. Given that the first removed (C) (D)
9 9
ball is white, the probability that the
1.14 A box contains 5 black balls and 3 red
second removed ball is red is
balls. A total of three balls are picked
1 3
(A) (B) from the box one after another, without
3 7 replacing them back. The probability of
1 4 getting two black balls and one red ball
(C) (D)
2 7 is
1.10 Let w be a complex cube rate of unity 3 2
(A) (B)
with w  1 . A fair die is thrown three 8 15
times. If r1 , r2 , r3 are the no. obtained on 15 1
the die then the probability that (C) (D)
28 2
wr1  wr2  wr3  0 1.15 A fair coin is tossed N times. The
2 1 probability that head does not turn up in
(A) (B)
9 9 any of the tosses is
N 1 N 1
3 1 1 1
(C) (D) (A)   (B) 1   
8 4 2 2
1.11 If a leap year is selected at random, what N N
1 1
is the chance it will contains 53 (C)   (D) 1   
Sundays. 2 2
1 1 1.16 A box contains 4 red balls and 6 black
(A) (B) balls. Three balls are selected randomly
2 7
from the box one after another, without
2 1
(C) (D) replacement. The probability that the
7 9
selected set contains one red ball and
1.12 Candidates were asked to come to an two black balls is
interview with 3 pens each. Black, blue,
1 1
green and red were the permitted pen (A) (B)
20 12
colours that the candidate could bring.
The probability that a candidate comes 3 1
(C) (D)
with all 3 pens having the same colour is 10 2
_____. 1.17 Two white and two black balls, kept in
1.13 A urn contains 5 red ball and 5 black two bins, are arranged in four ways as
balls. In the first draw, one ball is picked shown below. In each arrangement, a
at random and discarded without bin has to be chosen randomly and only
noticing its colour. The probability to one ball needs to be picked randomly
get a red ball in the second draw is from the chosen bin. Which one of the

 
Probability | 3 
following arrangements has the highest 1 550
(A) (B)
probability for getting a white ball 4 2652
picked? 550
(A) (C) (D) Not
2650
1.22 A box contains 3 red and 2 black balls.
Four balls are removed from the box one
(B) by one, without replacement. The
probability of the ball remaining in the
box being red, is
(C) 609 3
(A) (B)
625 5
2 81
(C) (D)
(D) 5 625
1.23 Three persons P, Q and R independently
try is hat a largest. If the probability of
1.18 Two dice are thrown simultaneously. 3 1 5
their ratting the largest are , and
The probability that the sum of numbers 4 2 8
on both exceeds 8 is, respectively. Then the probability that
4 7 the largest is hat by P or Q but not by R
(A) (B)
36 36 21 1
(A) (B)
9 10 64 64
(C) (D)
36 36 21 20
(C) (D)
1.19 Two bags contain ten coins each and the 60 64
coins in each bag are numbered from 1 1.24 A box containing 10 identical
to 10. One coin is drawn at random from compartments has 6 red balls and 2 blue
each bag. The probability that one of the balls. If each compartment can hold only
coins has value 1, 2, 3, or 4, while the one ball, then the number of different
other has value 7, 8, 9 or 10 is possible arrangements are
2 4 (A) 1026 (B) 1062
(A) (B)
5 25 (C) 1260 (D) 1620
8 16 1.25 For two rolls of a fair die, the
(C) (D)
25 25 probability of getting a 4 in the first
1.20 An integer is chosen at random 1 from roll and a number less than 4 in the
200 positive integers the probability that second roll, up to 3 digits after the
the integer chosen is divisible by 6 or 8. decimal point, is ____.
1.21 Two cards are drawn from a peak of 52 1.26 What is the probability of rolling A sum
cards what is the probability that either greater than 7 with two dice if it is
both are red or both are kings. known that the font die is a 4.

 
4 | Engineering Mathematics  
1 1 1.31 The probability that a communication
(A) (B)
8 2 system will have high fidelity is 0.81.
1 1 The probability that the system will
(C) (D) have both high fidelity and high
4 16
selectivity is 0.18. The probability that a
1.27 In rolling of two fair dice, the outcome given system with high fidelity will
of an experiment is considered to be the have high selectivity is
sum of the numbers appearing on the
(A) 0.181 (B) 0.191
dice. The probability is highest for the
outcome of _____. (C) 0.222 (D) 0.826
1.28 Let E and K be two independent events 1.32 Two bags A and B have equal number of
the probability that both E and F balls. Bag A has 20% red balls and 80%
1 green balls. Bag B has 30% red balls,
Happens is and the neither E nor F 60% green balls and 10% yellow balls.
12
1 P( E ) Contents of Bags A and B are mixed
happens is then a value of thoroughly and a ball is randomly
2 P( F )
picked from the mixture. What is the
4 3 chance that the ball picked is red?
(A) (B)
5 2
(A) 20% (B) 25%
1 (C) 30% (D) 40%
(C) (D) Not
5
1.33 A bag contains 4 tickets no. 1, 2, 3, 4
1.29 Consider a sequence of tossing of a fair and another bag contains 6 tickets no. 2,
coin where the outcomes of tosses are 4, 6, 7, 8, 9 one bag is chosen and
independent. The probability of getting tickets is dr5awn the probability that the
the head for the third time in the fifth tickets bears the no. 4 is
toss is
5 1
5 3 (A) (B)
(A) (B) 24 24
16 16
2
3 9 (C) (D) Not
(C) (D) 5
5 16
1.34 A box contains three blue balls and four
1.30 Let S be the sample space and two red balls. Another identical box contains
mutually exclusive events A and B two blue balls and five red balls. One
A  B  S . If P denotes the probability ball is picked at random from one of the
of the event the maximum value of two boxes and it is red. The probability
P [ A] P [ B ]  ? that it came from the first box is
1 1 2 4
(A) (B) (A) (B)
9 2 3 9
1 3 4 2
(C) (D) (C) (D)
4 4 7 7
 
Probability | 5 
1.35 In an entrance test are MCIS, there are 4 received by station A and then
possible answers to each question of transmitted to station B. The probability
which one is correct. The probability of each station receiving the signal
that a student knows the answer to the 3
correctly is . If the signal receiver at
question is 90 %. If the gest the correct 4
answer to the question then the station B is given then the probability
probability that he was guessing is that the original signal was green.
1 1 12 20
(A) (B) (A) (B)
37 22 23 23
1 1
(C) (D) 22
38 4 (C) (D) None
23
1.36 A box contains 2 washers, 3 nuts and 4
bolts. Items are drawn from the box at 1.39 1 ,  2 ,....,  n are n keys on an ancient
random one at a time without keyboard and the probability that the
replacement. The probability of drawing correct letter is printing on pressing a
2 washers first followed by 3 nuts and key in P, A key is selected at random
subsequently the 4 bolts is and pressed twice. Both times 1 is
2 1 printed. Find the probability that the
(A) (B)
315 630 selected key was 1 .
1 1
(C) (D) 1.40 A man is known to speak truth 3 out of
1260 2520
4 times. He throws a die and report it is
1.37 Ley U 1 and U 2 be two wins such that
6, the probability it is actually 6.
U 1 contains 3 W and 2 R balls and U 2
1 3
contains only 1 W ball. A fair coin is (A) (B)
2 4
tossed if head appears then one ball is
drawn at random from U 1 and put into 3
(C) (D) Not
U 2 . However, if tail appears then 2 balls
8

are drawn at random from U 1 and put 1.41 A man is known to speak truth 3 out of
4 times. He knows A die and repost. It is
into U 2 . Now 1 ball is drawn at random
six. Find the probability that its actually
from U 2 . Find A six.
(i) The probability of the drawn fall 1.42 A bag contains 6 white and unknown
from U 2 being white. no. of black balls ( 3) balls are drawn
(ii) Given that the drawn ball from U 2 is one by one with replacement from his
white. The probability that head bag twice and is found to be white on
appeared on coin. both occasion. Find the probability that
1.38 A signal which can be G or R with the bag has exactly 3 black balls.
4 1 1.43 A pair of fair dice is rolled
probability and respectively is
5 5 simultaneously.

 
6 | Engineering Mathematics  
The probability that the sum of the respectively. The values of P and E[ X ]
numbers from the dice equals six is are respectively
1 7
(A) (B) (A) 0.05, 1.87 (B) 1.90, 5.87
6 36
(C) 0.05, 1.10 (D) 0.25, 1.40
5 1
(C) (D) 1.48 A random variable X has probability
36 12
1.44 A fair die is rolled four times. Find the density function f ( x) as given below :
probability that six shows up twice  a  bx for 0  x  1
1 16 f ( x)  
(A) (B)  0 otherwise
2 325
2
1 25 If the expected value E[ X ]  , then
(C) (D) 3
36 216
1.45 If X is a continuous random variable P [ X  0.5] is ______.
whose probability density function is 1.49 Two players, A and B alternately keep
given by rolling a fair dice. The person to get a
 K (5 x  2 x 2 ), 0  x  2 six first wins the game. Given that
f ( x)  
0, otherwise player A starts the game, the probability

that A wins the game is
Then P( X  1) is
3 4 5 1
(A) (B)
(A) (B) 11 2
14 5
14 17 7 6
(C) (D) (C) (D)
17 28 13 11
1.46 Consider the function f ( x) as a pdf for 1.50 Consider the following probability mass
the continuous random variable X function (p.m.f) of a random variable X
 x  q, if X  0
 4 0 x2

 p ( X , q )  1  q, if X  1
4  x  0,
f ( x)   2 x4  otherwise
 4
 0 Other value of x If q  0.4 , the variance of X is____.

 1.51 The graph of a function f ( x) is shown
(A) Show that f ( x) is a valid pdf in the figure
(B) Find P( X  1) f ( x)
(C) Find P ( X  3)
3h
(D) Find P(1  X  3)
(E) Find P( X  1 or X  3) 2h
1.47 The random variable X takes on the
values 1, 2, or 3 with probabilities h
(2  5P) (1  3P) (1.5  2 P)
, and
5 5 5 x
0 1 2 3

 
Probability | 7 
For f ( x) to be a valid probability two stations. Until they are successful if
density function, the value of h is there is 40 % chance of getting
(A) 1/3 (B) 2/3 reservation in any attempt by a
passenger then the average no. of
(C) 1 (D) 3
attempts that passenger need to make to
1.52 A fair die with faces [1, 2, 3, 4, 5, 6] is get a seat reserved.
thrown repeatedly till ‘3’ is observed for
(A) 2.5 (B) 3.5
the first time. Let X denote the number
of times the die is thrown. The expected (C) 3.0 (D) 2.2
value of X is ______. 1.59 If E (2 X  Y )  0
1.53 Let the random variable X represents no. E ( X  2Y )  33
of times A fair coin need to be Jessed till Then the value of E [ X ]  E [Y ]  ?
two consecutive heads appear for the
1.60 The variance of the random variable
first time the repeated value of X.
with pdf
3 5
(A) (B) 1 x
2 2 f ( x)  xe
2
1
(C) (D) Not 1.61 The variance of a random variable X is
2
 2X . Then the variance of kx
1.54 Can a random variable how negative
output. (A) k 22X (B) k 2X
1.55 Consider a dice with the property that (C)  k  X (D) k 2 2X
the probability of a face with n dots
1.62 The probability density function of a
showing up is proportional to n. The
random variable X is
probability of the face with three dots
showing up is ______. e  x x0
f ( x)  
1.56 A random variable X has following pdf 0 Otherwise
2
1  ( x81) He expected value of the function
f ( x)  e   x   3x
8 g ( x)  e 4

(A) 2 (B) 4
Then the value of  f ( x) dx
1 (C) 3 (D) 0
1 1 1.63
(A) (B)
2 4 P( X = x)
(C) 1 (D) 
1/4
1.57 If the SD of the binomial distribution
( q  p )16 is 2 then the mean of the
–1 0 1 2 3
distribution. For a random variable X following the
1.58 Passengers try repeatedly to get a seat pdf as shown in figure. The values of
reservation in any train running between mean and the variance are respectively.

 
8 | Engineering Mathematics  
1.64 The figure shows the schematic of a x

production process with machines A, B e2


function , the value of the constant
and C. An input job needs to be pre- K
processed either by A or by B before it is K is _______.
fed to C, from which the final finished 1.69 Consider a Gaussian distributed random
product comes out. The probabilities of variable with zero mean and standard
failure of the machines are given as, deviation  . The value of its cumulative
P( A)  0.15 , P( B )  0.05, P (C )  0.1 distribution function at the origin will be
(A) 0 (B) 0.5
A
Input
(C) 1 (D) 10
Finished
C
job product 1.70 The box 1 contains chips numbered 3, 6,
B 9, 12 and 15. The box 2 contains chips
Assuming independence of failures of numbered 6, 11, 16, 21 and 26. Two
the machines, the probability that a chips, one from each box, are drawn at
given job is successfully processed (up random. The numbers written on these
to the third decimal place) is _______. chips are multiplied. The probability for
1.65 A probability density function is of the the product to be an even number is
form p ( x )  Ke  a| x | , x  ( , ) . The 6 2
(A) (B)
value of K is 25 5
(A) 0.5 (B) 1 3 19
(C) (D)
(C) 0.5a (D) a 5 25
1.71 In an examination of 10 multiple choice
1.66 A continuous random variable X has a
questions (1 or more can be correct out
probability density function
of 4 options). A student deudes to mark
f ( x)  e x , 0  x   .
the answers at random. Find the
Then P{ X  1} is probability that he gets exactly two
(A) 0.368 (B) 0.5 questions correct.
(C) 0.632 (D) 1.0 1.72 The diameters of 10000 ball bearings
1.67 PX ( x)  Me2| x |  Ne3| x | were measured. The mean diameter and
standard deviation were found to be 10
is the probability density function for
mm and 0.05 mm respectively.
the real random variable X over the
Assuming Gaussian distribution of
entire x-axis. M and N are both positive
measurements, it can be expected that
real numbers. The equation relating M
the number of measurements more than
and N is
10.15 mm will be
2 1 (A) 230 (B) 115
(A) M  N  1 (B) 2M  N  1
3 3 (C) 15 (D) 2
(C) M  N  1 (D) M  N  3 1.73 Probability density function differential
1.68 Given that X is a random variable in the function p( x) of a random variable x is
range  0, with a probability density as shown below. The value of  is

 
Probability | 9 
p( x) 1.77 How many times must a man ton a fair
coin so that the probability of having at
least one head is more than 90 %.
90
P{Atleast one head} 
x 100
0 a a+b a+c
(A) 6 (B) 8
2 1 (C) 3 (D) 4
(A) (B)
c c
1.78 Difference between mean and variance
2 1 of a bionomical vacate is 1 and
(C) (D)
(b  c ) (b  c ) difference between their squares is 11.
1.74 A probability distribution with right Find probability of getting exactly three
skew is shown in the figure. sources.
f ( x)  1 5
p q
 6 6
 n  36

1.79 The probability density function of


evaporation E on any day during a year
x
in a watershed is given by
The correct statement for the probability
distribution is 1
 ; 0  E  5 mm/day
(A) Mean is equal to mode. f (E)  5
 0; otherwise
(B) Mean is greater than median but less
than mode. The probability that E lies in between 2
(C) Mean is greater than median and and 4 mm/day in the watershed is (in
mode. decimal) ___.
(D) Mode is greater than median. 1.80 A fair (unbiased) coin was tossed four
1.75 The mean and variance of binomial times in succession and resulted in the
distribution are 4 and 2 rest then the following outcomes :
probability of two successes. (i) Head (ii) Head (iii) Head (iv) Head.
7 2 The probability of obtaining a ‘Tail’
(A) (B)
64 3 when the coin is tossed again is
5 1 1
(C) (D) (A) 0 (B)
9 64 2
1.76 An unbiased coin is tossed an infinite 4 1
(C) (D)
number of times. The probability that 5 5
the fourth head appears at the tenth toss 1.81 If X is a continuous, real valued random
is variable defined over the interval
(A) 0.067 (B) 0.073 ( , ) and its occurrence is defined
(C) 0.082 (D) 0.091 by the density function given as,

 
10 | Engineering Mathematics  
1 x a 
2
H. If the coin is tossed one more time,
1  
f ( x)  e2 b 
the probability (up to one decimal place)
2  b
of obtaining H again, given the previous
where ‘a’ and ‘b’ are the statistical realizations of H, H and H, would be
attributes of the random variable X. The _____.
value of the integral
1.86 A random variable is uniformly
2
a 1 x  a 
1   distributed over the interval 2 to 10. Its

2 b 
e dx is
 2  b variance will be
(A) 1 (B) 0.5 16
(A) (B) 6
 3
(C)  (D) 256
2 (C) (D) 36
1.82 For probability density function of a 9
random variable, x is 1.87 X is uniformly distributed random
x variable that take values between 0 and
f ( x)  (4  x 2 ) ; for 0  x  2 1. The value of E ( X 3 ) will be,
4
0 ; otherwise 1
(A) 0 (B)
The mean  x of the random variable is 8
______. 1 1
(C) (D)
1.83 Probability density function of a random 4 2
variable X is given below 1.88 If f ( x) and g ( x) are two probability
0.25; if 1  x  5 density functions,
f ( x)  
 0 ; otherwise  x
 a 1 ;  a  x  0
P ( X  4) is 
 x
3 1 f ( x)    1 ; 0  x  a
(A)
4
(B)
2  a
 0 ; otherwise
1 1 
(C) (D) 
4 8
 x
1.84 X and Y are two random independent  a ; a  x  0
events. It is known that P( X )  0.40 
 x
and P ( X  Y C )  0.7. Which one of the g ( x)   ; 0 xa
 a
following is the value of P ( X  Y ) ?  0 ; otherwise

(A) 0.7 (B) 0.5 
(C) 0.4 (D) 0.3 Which one of the following statement is
1.85 A two-faced fair coin has its faces true?
designated as head (H) and tail (T). This (A) Mean of f ( x) and g ( x) are same;
coin is tossed three times in succession Variance of f ( x) and g ( x) are
to record the following outcomes : H, H,
same

 
Probability | 11 
(B) Mean of f ( x) and g ( x) are same; 1.92 Let X 1, X 2 , X 3 are theses random
Variance of f ( x) and g ( x) are variable with uniform listen function
different find the probability of
P ( X 1 is largest)
(C) Mean of f ( x) and g ( x) are
different; Variance of f ( x) and 1 2
(A) (B)
3 3
g ( x) are same
1
(D) Mean of f ( x) and g ( x) are (C) (D) Not
6
different; Variance of f ( x) and 1.93 The number of parameters in the
g ( x) are different univariate exponential and Gaussian
distributions, respectively are
1.89 The pdf of a Gaussian random variable
(A) 2 and 2 (B) 1 and 2
X is given by
(C) 2 and 1 (D) 1 and 1
2
 ( x  )
1 1.94 Let X 1 , X 2 , X 3 and X 4 be independent
PX ( x )  e 18
3 2 normal random variables with zero
The probability of the event P ( X  4) mean and unit variance. The probability
that X 4 is the smallest among the four is
is
________.
1
(A) (B) 0 1.95 E1 and E2 are events in a probability
2
space satisfying the following constants
1 1 P ( E1 )  P ( E2 ) , P ( E1  E2 )  1 , E1 and
(C) (D)
3 2 4 E2 are independent then P ( E1 )  ______.
1.90 Subway trains on a certain lines run – 1
every half hour between midnight and 6 (A) 0 (B)
4
in the morning. What is the probability 1
that A man interning the station at a (C) (D) 1
2
random time during this period will 1.96 Consider two events E1 and E2 such
have to wait atleast 20 minutes.
1 1
1 1 that P( E1 )  , P( E2 )  and
(A) (B) 2 3
3 4 1
P( E1  E2 )  . Which of the following
5 5
(C) (D) 0
6 statement is true?
1.91 For the function f ( x)  a  bx , 0  x  1 2
(A) P( E1  E2 ) 
to be a valid probability density 3
function, which one of the following (B) E1 and E 2 are independent
statements is correct? (C) E1 and E 2 are not independent
(A) a  1 , b  4 (B) a  0.5 , b  1 E  4
(D) P  1  
(C) a  0, b  1 (D) a  1, b  1  E2  5

 
12 | Engineering Mathematics  
1.97 The probability that two friends share 1.102 Assume mean hl. Of soldiers to be 68.22
the same birth-month is inches with a variance pf 10.8 inches.
1 1 How many soldiers in a regiment of
(A) (B)
6 12 1000 would you expect to be over 6 ft
1 1 tall. Given that the standard normal
(C) (D) curve between x  0 and
144 24
1.98 Let P(E) denote the probability of an x  0.35 is 0.1368 and x  0 to
1 x  1.15 is 0.3746
event E. Given P(A) = 1, P( B)  , the
2 (A) 112 (B) 120
 A B (C) 121 (D) 125
values of P  and P 
B  A 1.103 A pie is rolled 180 times using the
respectively are normal distribution find the problem of
1 1 1 1 a face 4 will turn up at least 35 times the
(A) , (B) ,
4 2 2 4 area under the normal curve Z  0 to
1 1 Z 1
(C) ,1 (D) 1,
2 2 (A) 0.5 (B) 0.10
1.99 A box contains 5 black and 5 red balls. (C) 0.3174 (D) 0.1587
Two balls are randomly picked one after
1.104 Let X 1 , X 2 , X 3 be independent and
another from the box, without
replacement. The probability for both identically distributed RV with uniform
balls being red is [0, 1]
distribution on a b . The probability
1 1
(A) (B)
90 2 P[ X 1  X 2  X 3 ] .
19 2 1
(C) (D) (A) 0.1587 (B)
90 9 2
1.100 A lot has 10% defective items. Ten (C) 0.5 (D) 1.2
items are chosen randomly from this lot.
1.105 Let X be a zero mean unit variance
The probability that exactly 2 of the
chosen items are defective is Gaussian random variable. E  x  is
(A) 0.0036 (B) 0.1937 equal to _____.
(C) 0.2234 (D) 0.3874 1.106 A box contains 20 defective items and
1.101 A nationalized bank has found that the 80 non-defective items. If two items are
daily balance available in its savings selected at random without replacement,
accounts follows a normal distribution what will be the probability that both
with a mean of Rs. 500 and a standard items are defective?
deviation of Rs. 50. The percentage of 1 1
(A) (B)
savings account holders, who maintain 5 25
an average daily balance more than Rs. 20 19
(C) (D)
500 is _______. 99 495

 
Probability | 13 
1.107 A single die is thrown twice. What is the replacement. The probability that the
probability that the sum is neither 8 nor selected set contains one red ball and
9? two black balls is
1 5 1 1
(A) (B) (A) (B)
9 36 20 12
1 3 3 1
(C) (D) (C) (D)
4 4 10 2
1.108 Consider the continuous random 1.112 The standard deviation of a uniformly
variable with probability density distributed random variable between 0
function and 1 is
f (t )  1  t for 1  t  0 1 1
(A) (B)
 1  t for 0  t  1 12 3
The standard deviation of the random 5 7
(C) (D)
variable is 12 12
1 1 1.113 If three coins are tossed simultaneously,
(A) (B)
3 6 the probability of getting at least one
1 1 head is
(C) (D)
3 6 1 3
(A) (B)
1.109 Let X and Y be two independent random 8 8
variables. Which one of the relations 1 7
between expectation (E), variance (Var) (C) (D)
2 8
and covariance (Cov) given below is 1.114 The probability that a student knows the
False? correct answer to a multiple choice
(A) E ( XY )  E ( X ) E (Y ) question is 2/3. If the student does not
(B) Cov( X , Y )  0 know the answer, then the student
(C) Var( X  Y )  Var( X )  Var(Y ) guesses the answer. The probability of
the guessed answer being correct is 1/4.
(D) E ( X 2Y 2 )   E  X    E Y  
2 2
Given that the student has answered the
1.110 A coin is tossed 4 times. What is the question correctly, the conditional
probability of getting heads exactly 3 probability that the student knows the
times? correct answer is
1 3 2 3
(A) (B) (A) (B)
4 8 3 4
1 3 5 8
(C) (D) (C) (D)
2 4 6 9
1.111 A box contains 4 red balls and 6 black 1.115 Let X be a normal random variable with
balls. Three balls are selected randomly mean 1 and variance 4. The probability
from the box one after another, without P  X  0 is

 
14 | Engineering Mathematics  
(A) 0.5 1.120 In the following table, X is a discrete
(B) greater than zero and less than 0.5 random variable and P( x) is the
probability density. The standard
(C) greater than 0.5 and less than 1.0
deviation of X is
(D) 1.0
x 1 2 3
1.116 Out of all the 2-digit integers
P(x) 0.3 0.6 0.1
between 1 and 100, a 2-digit number
has to be selected at random. What (A) 0.18 (B) 0.36
is the probability that the selected
(C) 0.54 (D) 0.6
number is not divisible by 7?
1.121 A batch of one hundred bulbs is
13 12
(A) (B) inspected by testing four randomly
90 90
chosen bulbs. The batch is rejected if
78 77 even one of the bulbs is defective. A
(C) (D)
90 90 batch typically has five defective bulbs.
1.117 A group consists of equal number of The probability that the current batch
men and women. Of this group 20% of accepted is ______.
the men and 50% of the women are 1.122 Among the four normal distribution with
unemployed. If a person is selected at probability density functions as shown
random from this group, the probability below, which one has the lowest
of the selected person being employed is variance?
________.
IV
1.118 Consider an unbiased cubic dice with
III
opposite faces coloured identically and II
each face coloured red, blue or green I

such that each colour appears only two


times on the dice. If the dice is thrown
thrice, the probability of obtaining red –2 –1 0 1 2
colour on top face of the dice at least (A) I (B) II
twice is ________.
(C) III (D) IV
1.119 A box contains 25 parts of which 10 are
1 1
defective. Two parts are being drawn 1.123 If P( X )  P(Y )  and P( X  Y ) 
4 3
simultaneously in a random manner
from the box. The probability of both 1 Y 
, the value of P   is
the parts being good is 12 X
7 42 1 4
(A) (B) (A) (B)
20 125 4 25
25 5 1 29
(C) (D) (C) (D)
29 9 3 50

 
Probability | 15 
1.124 The probability that a screw 1.129 The annual precipitation data of a city is
manufactured by a company is defective normally distributed with mean and
is 0.1. The company sells screws in standard deviation as 1000 mm and 200
packets containing 5 screws and gives a mm, respectively. The probability that
guarantee of replacement if one or more the annual precipitation will be more
screws in the packet are found to be than 1200 mm is
defective. The probability that a packet (A) < 50 % (B) 50 %
would have to be replaced is ________. (C) 75 % (D) 100 %
1.125 A hydraulic structure has four gates 1.130 In an experiment, positive and negative
which operate independently. The values are equally likely to occur. The
probability of failure of each gate is 0.2. probability of obtaining at most one
Given that gate 1 has failed, the negative value in five trials is
probability that both gates 2 and 3 will 1 2
(A) (B)
fail is 32 32
(A) 0.240 (B) 0.200 3 6
(C) (D)
(C) 0.040 (D) 0.008 32 32
1.131 Events A and B are mutually exclusive
1.126 A box contains 10 screws, 3 of which
and have nonzero probability. Which of
are defective. Two screws are drawn at
the following statement (s) are true?
random with replacement. The
(A) P( A  B )  P( A)  P( B )
probability that none of the two screws
is defective will be (B) P ( B C )  P ( A)
(A) 100% (B) 50% (C) P ( A  B )  P( A) P ( B )
(C) 49% (D) none of these (D) P ( B C )  P ( A)
1.127 If probability density function of a 1.132 The function shown in figure can
random variable x is represent a probability density function
for A _____.
 x 2 , 1  x  1
f ( x)   f ( x)
 0, elsewhere
3A
Then, the percentage probability
 1 1 2A
P  x   is A
 3 3
(A) 0.274 (B) 2.47 x
–3 –2 –1 0 1 2 3
(C) 24.7 (D) 247 1.133 A fair dice is rolled twice. The
1.128 Find the value of  such that function probability that an odd number will
f ( x) is valid probability density follow an even number is
function 1 1
(A) (B)
2 6
f ( x)    x  1 2  x  ; for 1  x  2
1 1
(C) (D)
=0 ; otherwise 3 4
 
16 | Engineering Mathematics  
1.134 If E denotes expectation, the variance of remaining half have two children per
a random variable X is given by family. The probability that a child
(A) E[ X 2 ]  E 2 [ X ] picked at random, has a sibling is
______.
(B) E[ X 2 ]  E 2 [ X ]
1.140 Parcels from sender S to receiver R pass
(C) E [ X 2 ]
sequentially through two post-offices.
(D) E 2 [ X ] 1
Each post-office has a probability of
1.135 An examination consists of two papers, 5
paper 1 and paper 2. The probability of losing an incoming parcel, independently
failing in paper 1 is 0.3 and that in paper
of all other parcels. Given that a parcel
2 is 0.2. Given that a student has failed
is lost, the probability that it was lost by
in paper 2, the probability of failing in
the second post-office is________.
paper 1 is 0.6. The probability of a
student failing in both papers is 1.141 Three fair cubical dice are thrown
(A) 0.5 (B) 0.18 simultaneously. The probability that all
three dice have the same number of dots
(C) 0.12 (D) 0.06
on the faces showing up is (up to third
1.136 A fair coin is tossed 10 times. What is
decimal place) _______.
the probability that only the first two
tosses will yield heads? 1.142 Suppose A and B are two independent
2 2 events with probabilities P( A)  0 and
1 1
10
(A)   (B) C2   P( B)  0 .
2 2
10 10 Let A and B be their complements.
1 1
10
(C)   (D) C2   Which one of the following statements
2 2
is FALSE?
1.137 A fair coin is tossed independently four
(A) P ( A  B )  P( A) P ( B )
times. The probability of the event “the
number of time heads shown up is more (B) P ( A / B )  P ( A)
than the number of times tails shown
up” is (C) P( A  B )  P( A)  P( B )
1 1 (D) P ( A  B )  P ( A ) P ( B )
(A) (B)
16 8
1.143 A fair coin is tossed three times in
1 5
(C) (D) succession. If the first toss produces a
4 16
head, then the probability of getting
1.138 A fair coin is tossed repeatedly till both exactly two heads in three tosses is
head and tail appear at least once. The
average number of tosses required is 1 1
(A) (B)
_____. 8 2
1.139 In a housing society, half of the families 3 3
(C) (D)
have a single child per family while the 8 4

 
Probability | 17 
1.144 If P and Q are two random events, then If three identical dice as the above are
which of the following is true? thrown, the probability of occurrence of
(A) Independence of P and Q implies values 1, 5 and 6 on the three dice is
that Probability ( P  Q )  0 (A) Same as that of occurrence of 3, 4, 5.
(B) Same as that of occurrence of 1, 2, 5.
(B) Probability ( P  Q )  Probability (P)
1
+ Probability (Q) (C)
128
(C) If P and Q are mutually exclusive
5
then they must be independent (D)
8
(D) Probability ( P  Q )  Probability (P) 1.148 A zero mean random signal is uniformly
1.145 The probability of getting a “head” in a distributed between limits – a and + a
single toss of a biased coin is 0.3. The and its mean square value is equal to its
coin is tossed repeatedly till a “head” is variance. Then the r.m.s value of the
obtained. If the tosses are independent, signal is
then the probability of getting “head” a a
(A) (B)
for the first time in the fifth toss is 3 2
______.
(C) a 2 (D) a 3
1.146 Assume for simplicity that N people, all
1.149 A fair coin is tossed till a head appears
born in April (a month of 30 days) are
for the first time. The probability that
collected in a room, consider the event
the number of required tosses is odd, is
of at least two people in the room being
1 1
born on the same date of the month even (A) (B)
3 2
if in different years e.g. 1980 and 1985.
What is the smallest N so that the 2 3
(C) (D)
probability of this exceeds 0.5 is? 3 4
(A) 20 (B) 7 1.150 An unbiased coin is tossed five times.
The outcome of each toss is either a
(C) 15 (D) 16
head or a tail. The probability of getting
1.147 A loaded dice has following probability at least one head is
distribution of occurrences 1 13
(A) (B)
Dice 32 32
Probability
value 16 31
(C) (D)
1 1/4 32 32
2 1/8
3 1/8
4 1/8
5 1/8
6 1/4

 
18 | Engineering Mathematics  

Answer Key : Probability


1.1 A 1.2 A 1.3 * 1.4 A 1.5 C
1.6 A 1.7 D 1.8 A 1.9 C 1.10 A
1.11 C 1.12 0.01818 1.13 A 1.14 C 1.15 C
1.16 D 1.17 C 1.18 D 1.19 C 1.20 1/4
1.21 B 1.22 B 1.23 A 1.24 C 1.25 0.083
1.26 B 1.27 7 1.28 * 1.29 B 1.30 C
1.31 C 1.32 B 1.33 A 1.34 B 1.35 *
1.36 C 1.37 * 1.38 B 1.39 * 1.40 *
1.41 * 1.42 * 1.43 C 1.44 D 1.45 D
1.46 * 1.47 A 1.48 0.25 1.49 D 1.50 0.24
1.51 A 1.52 6 1.53 A 1.54 * 1.55 0.142
1.56 A 1.57 * 1.58 A 1.59 * 1.60 6
1.61 A 1.62 B 1.63 * 1.64 0.893 1.65 C
1.66 A 1.67 A 1.68 2 1.69 B 1.70 D
1.71 * 1.72 C 1.73 A 1.74 C 1.75 *
1.76 C 1.77 D 1.78 * 1.79 A 1.80 B
1.81 B 1.82 1.066 1.83 A 1.84 A 1.85 0.5
1.86 A 1.87 C 1.88 B 1.89 * 1.90 *
1.91 B 1.92 A 1.93 B 1.94 0.25 1.95 D
1.96 C 1.97 B 1.98 D 1.99 D 1.100 B
1.101 50 1.102 * 1.103 D 1.104 * 1.105 0.7978
1.106 D 1.107 D 1.108 B 1.109 D 1.110 A
1.111 D 1.112 A 1.113 D 1.114 D 1.115 B
1.116 D 1.117 0.65 1.118 0.259 1.119 A 1.120 D
1.121 0.8145 1.122 D 1.123 C 1.124 0.4095 1.125 C
1.126 C 1.127 B 1.128 6 1.129 A 1.130 D
1.131 A 1.132 0.125 1.133 D 1.134 A 1.135 C
1.136 C 1.137 D 1.138 C 1.139 0.66 1.140 0.444
1.141 0.027 1.142 C 1.143 B 1.144 D 1.145 0.072
1.146 B 1.147 C 1.148 A 1.149 C 1.150 D



 
 
 
 

2.1 The value of the integral (A) is – 1.


1    x2  (B) is 0.
2 0
I exp   dx is (C) is 1.
 8 
(D) depends on the direction (Clockwise
(A) 1 (B)  or anti-clockwise) of the semicircle.
(C) 2 (D) 2  2.5 The value of the integral of the function
 g ( x, y )  4 x 3  10 y 4  along the straight
2.2 The integral 
0
sin 3  d  is given by
line segment from the point (0, 0) to the
1 2 point (1, 2) in the x-y plane is
(A) (B)
2 3 (A) 33 (B) 35
4 8 (C) 40 (D) 56
(C) (D) 2
3 3 1  2x
2  
2.6 The integral e dx  is equal to
2.3 The following plot shows a function y
which varies linearly with x. The value 1 1
2
(A) (B)
of the integral I =  y dx is 2 2
1
y (C) 1 (D) 
3 2.7 The volume under the surface
z ( x, y )  x  y and above the triangle is
2 the x-y plane defined by { 0  y  x and
1 0  x  12 } is _______.
x 2.8 The value of the integral
–1 0 1 2 3 
sin(4t )
(A) 1.0 (B) 2.5  12cos(2t )

4t
dt is _______.
(C) 4.0 (D) 5.0
2.9 The contour on the xy plane, where the
2.4 Consider points P and Q in the x-y
partial derivative of x 2  y 2 with respect
plane, with P  (1, 0)  and Q  (0,1) .
Q
to y is equal to the partial derivative of
The line integral 2  ( xdx  ydy ) along 6 y  4 x with respect to x is
P

the semicircle with the line segment PQ (A) y  2 (B) x  2


as its diameter (C) x  y  4 (D) x  y  0

 
2 | Engineering Mathematics  

ax 2  by 2 2.16 If S   x 3dx , then S has the value
2.10 Let f ( x, y )  , where a and b 1
xy
1 1
f f (A) (B)
are constants. If  at x  1 and 3 4
x y
1
y  2 , then the relation between a and b (C) (D) 1
2
is 2.17 f (x, y) is a continuous function defined
b b over (x, y)   0,1   0,1 . Given the two
(A) a  (B) a 
4 2
(C) a  2b (D) a  4b constraints, x  y 2 and y  x 2 , the
2.11 Let r  x 2  y  z and z 3  xy  yz volume under f (x, y) is
y 1 x  y
 y 3  1 . Assume that x and y are
independent variables. At ( x, y, z )
(A)   f  x, y  dxdy
y 0 x  y 2

 (2, 1,1) , the value (correct to two y 1 x 1

r
(B)   f  x, y  dxdy
decimal places) of is _______. y  x2 x y 2
x y 1 x 1
2.12 The values of the integrals (C)   f  x, y  dxdy      
1 1 x  y  y 0 x  0
    0  0 ( x  y )3 dy  dx y  x x y

1  1 x y 
  (D)   f  x, y  dxdy
and    dx  dy are y 0 x 0
0 ( x  y )3
0
  2.18 The value of the quantity, where
(A) same and equal to 0.5. 1
P   xe x dx
(B) same and equal to – 0.5. 0

(C) 0.5 and – 0.5, respectively. (A) 0 (B) 1


(D) – 0.5 and 0.5, respectively. 1
(C) e (D)
1 e
2  D
2.13 The integral ( x  y  10) dx dy ,
2.19 A function y  5 x  10 x is defined over
2

where D denotes the disc : x 2  y 2  4 , an open interval x  (1, 2) . At least at


evaluates to _______. dy
one point in this interval, is exactly
2.14 A three dimensional region R of finite dx
volume is described by x 2  y 2  z 3 ; (A) 20 (B) 25
0  z  1 where x, y, z are real. The (C) 30 (D) 35
volume of R (up to two decimal places) 2.20 A particle, starting from origin at t  0s ,
______. is traveling along x -axis with velocity
2.15 Let I   (2 z dx  2 y dy  2 x dz ) , where   
v  cos  t  m/s .
C
2 2 
x, y, z are real, and let C be the straight At t  3 s, the difference between the
line segment from point A : (0, 2, 1) to distance covered by the particles and the
point B : (4, 1, –1). The value of I is magnitude of displacement from the
______. origin is______.

 
Calculus | 3 
2.21 To evaluate the double integral 2.25 The area bounded by the parabola
8  ( y /2)1  2x  y   2 y  x 2 and the lines x  y  4 is equal
  
0 y /2 
 2  
 dx  dy , to
(A) 6 (B) 18
 2x  y 
we make the substitution u    (C)  (D) None
 2 
x 2
dy
y
and v  . The integral will reduce to 2.26 Given y   cos t dt then is
2
1 dx

  2u du  dv
4 2 x2 x2
(A)  (A) 2 cos (B) 2 x cos
0 0 2 2

(B)    2u du  dv
4 1 (C) 2 x cos x 2 (D) x 2 cos x
0 0 2.27 The curve given by the equation

(C)    u du  dv
4 1 x 2  y 2  3axy is,
0 0
(A) Symmetrical about x-axis.
(D)    u du  dv (B) Symmetrical about y-axis.
4 2

0 0
(C) Symmetrical about the line y  x .
2.22 Let I  C  xy 2 dx dy , where R is the a
R (D) Tangential to x  y  .
region shown in the figure and 3
C  6  10 4 . The value of I equals x2 d
2.28 If ( x)   t dt , then is
________. (Give the answer up to two 0 dx
decimal places.) (A) 2x 2 (B) x
y
(C) 0 (D) 1
10  

R
2.29  
0
2
0
2
sin( x  y) dx dy

2 (A) 0 (B) 
x 
5 1 (C) (D) 2
2
2.23 Consider a function f ( x , y , z ) given by
2.30 The area enclosed between the parabola
f ( x, y, z )  ( x 2  y 2  2 z 2 )( y 2  z 2 ) .  y  x 2 and the straight line y  x is
The partial derivative of this function
(A) 1/8 (B) 1/6
with respect to x at the point, x  2 ,
(C) 1/3 (D) 1/2
y  1 and z  3 is ______.
2.31 If x  a (  sin ) and y  a (1  cos )
1

 dy
3
2.24 The value of e y y dy is
2
then is
0
dx
1 1  
(A)  (B)  (A) sin (B) cos
2 3 2 2
  
(C) (D) 3  (C) tan (D) cot
2 2 2

 
4 | Engineering Mathematics  
2.32 The volume of the object expressed in 2.37 The area enclosed between the curves
spherical co-ordinates is given by y 2  4 x and x 2  4 y is
2

1 16
V  (A) (B) 8
0  
0
3
0
r 2 sin  drd d  3
32
The value of the integral is (C) (D) 16
3
 
(A) (B) 2.38 The parabolic arc y  x , 1  x  2 is
3 6
revolved around the x-axis. The volume
2  of the solid of revolution is
(C) (D)
3 4 (A)  / 4 (B)  / 2
a (C) 3 / 4 (D) 3 / 2
2.33  a
(sin 6 x  sin 7 x) dx is equal to
2.39 If f (x) is an even function and ‘a’ is a
a

(A) 2 sin x dx
a
6 positive real number, then  a
f ( x ) dx
0
equals
(B) 2 sin 7 x dx
a
(A) 0 (B) a
0 a
(C) 2a (D) 2 f ( x) dx
a 0
(C) 2 (sin 6 x  sin 7 x) dx 2.40 The area enclosed between the straight
0

(D) zero line y  x  and the parabola y  x 2 in the


x-y plane is
2.34 Assuming i  1 and t is a real (A) 1/6 (B) 1/4

(C) 1/3 (D) 1/2
number,  0
3 it
e dt is 2.41 The value of the definite integral
e

(A)
3 1
i (B)
3 1
i
1
x ln ( x) dx is
2 2 2 2 4 3 2 2 3 4
(A) e  (B) e 
1  3 1  3 9 9 9 9
(C)  i  (D)  i  1   2 3 4 4 3 2
2  2  2  2  (C) e  (D) e 
9 9 9 9
2 3/2 2.42 The value of the integral
2.35 The length of the curve y  x
2 ( x  1) sin( x  1)
2
3
between x = 0 and x = 1 is 0 ( x  1)2  cos( x  1) dx is
(A) 0.27 (B) 0.67 (A) 3 (B) 0
(C) 1 (D) 1.22 (C) – 1 (D) – 2
2 x
2.36 Which of the following integrals is 2.43 The value of the integral  0 0
e x y dy dx
unbounded? is,

 1 1 1 2
(A)  tan x dx (A) (e  1) (B) (e  1) 2
0
4
(B) 
0 x 1
2
dx 2 2
2
 1 1 2 1 1
e  
1
 (D)  (C) (e  e) (D)
x
(C) xe dx dx
0 0 1 x 2 2 e

 
Calculus | 5 
2.44 The volume generated by revolving the 
2.49 Value of the integral I   4 cos2 xdx is
area bounded by the parabola y 2  8 x 0

and the line x  2 about y axis is,  1  1


(A)  (B) 
128 8 4 8 4
(A)  1  1
5 (C)   (D)  
5 8 4 8 4
(B) 3 x
128
127
2.50 The value of   (6  x  y) dx dy is
0 0
(C)
5 (A) 13.5 (B) 27.0
(D) None of the above (C) 40.5 (D) 54.0
2.45 Area bounded by the curve y  x 2 and 2.51 The value of the following definite

lines x = 4 and y = 0 is given by 2
sin 2 x
(A) 64 (B)
64 integral  1  cos x dx

is,
3 2

128 128 (A) –2 ln2 (B) 2


(C) (D)
3 4 (C) 0 (D) (ln 2) 2
2.46 The function f ( x )  e x is 2.52 The value of the following improper
1
(A) Even integral is  0
x ln x dx
(B) Odd
1
(C) Neither even nor odd (A) (B) 0
4
(D) None of the above 1
(C) (D) 1
2.47 By reversing the order of integration 4
2 2x
2.53 What is the area common to the circles
 
0 x2
f ( x, y ) dy dx may be represented
r  a and r  2 a cos  ?
as ______ (A) 0.524 a 2 (B) 0.614 a 2
2 2x
(A)   f ( x, y ) dy dx (C) 1.047 a 2 (D) 1.228 a 2
0 x2

2 y
2.54 What is the value of the definite integral
(B)   f ( x, y ) dx dy a x

0 y
dx ?
4 y
0
x  ax
(C) 0
y
2
f ( x, y) dx dy
(A) 0 (B)
a
2x 2 2
(D)   f ( x, y ) dy dx (C) a (D) 2a
x2 0

2.48 The area bounded by the parabola 2.55 The value of  6 cos4 3 sin 3 6 d 
0
2 y  x 2 and the line x  y  4 is equal
1
to (A) 0 (B)
15
(A) 6 (B) 18 8
(C)  (D) None of these (C) 1 (D)
3

 
6 | Engineering Mathematics  
2.56 Choose the most appropriate equation 2.62 Consider the following definite integral
for the function drawn as a thick line, in 1
(sin 1 x)2
the plot below. I  dx
y 0 1  x2
The value of the integral is,
(2, 0)
x 3 3
(A) (B)
(0, –1) 24 12
3 3
(A) x  y  y (B) x   ( y  y ) (C) (D)
48 64

(C) x  y  y (D) x   ( y  y ) 2.63 The value of the integral  0
x cos 2 x.dx is
2.57 The area of the region bounded by the
2 2
parabola y  x 2  1 and the straight line (A) (B)
8 4
x  y  3 is
2
59 9 (C) (D)  2
(A) (B) 2
6 2 1
1
(C)
10
(D)
7 2.64 The value of the integral x
1
2
dx is
3 6
(A) 2 (B) does not exist
2.58 The angle of intersection of the curves
(C) – 2 (D) 
x 2  4 y and y 2  4 x at point (0, 0) is
2.65 The curves, for which the curvature  at
(A) 0 0 (B) 300 any point is equal to cos 3  , where  is
(C) 45 0 (D) 90 0 the angle made by the tangent at that
2.59 The area between the parabola x 2  8 y point with the positive direction of the x-
and the straight line y  8 is ______. axis, are (given   y "/ [1  ( y ') 2 ]3/2 ,

2.60 If f ( x )  2 x 7  3 x  5 , which of the where y ' and y " are the first and

following is a factor of f ( x ) ? second derivatives of y with respect to


x).
(A) ( x 3  8) (B) ( x  1)
(A) circles (B) parabolas
(C) (2 x  5) (D) ( x  1) (C) ellipses (D) hyperbolas
2.61 Let x be a continuous variable defined 

e
 x2  y 2
over the interval (  ,  ), and 2.66 The value of the integral e dxdy
0 0
x
f ( x )  e  x e . is
The integral g ( x )   f ( x) dx is equal (A) /2 (B) 
(C)  (D)  / 4
to
x x 2.67 The expression e  ln (x )
for x  0 is equal to
(A) e e (B) e  e
(A)  x (B) x
x
(C) e e (D) e  x (C) x  1 (D)  x 1

 
Calculus | 7 

4
 1  tan x 
2.68   1  tan x  dx evaluates to
0

(A) 0 (B) 1
1
(C) ln 2 (D) ln 2
2
a y
2.69 The double integral   f ( x, y ) dx dy is
0 0

equivalent to
x y
(A)   f ( x, y ) dx dy
0 0
 y
(B)   f ( x, y ) dx dy
0 x
a a

(C)   f ( x, y) dx dy
0 x
a a

(D)   f ( x, y ) dx dy
0 0

 
8 | Engineering Mathematics  

Answer Key : Calculus


2.1 A 2.2 C 2.3 B 2.4 B 2.5 A
2.6 C 2.7 864 2.8 3 2.9 A 2.10 D
2.11 4.5 2.12 C 2.13 20 2.14 0.785 2.15 – 11
2.16 C 2.17 A 2.18 B 2.19 B 2.20 2
2.21 B 2.22 0.99 2.23 40 2.24 B 2.25 B
2.26 C 2.27 C 2.28 A 2.29 D 2.30 B
2.31 C 2.32 A 2.33 A 2.34 A 2.35 D
2.36 D 2.37 A 2.38 D 2.39 D 2.40 A
2.41 C 2.42 B 2.43 B 2.44 B 2.45 A
2.46 D 2.47 C 2.48 C 2.49 B 2.50 D
2.51 A 2.52 C 2.53 D 2.54 B 2.55 B
2.56 B 2.57 B 2.58 D 2.59 85.33 2.60 B
2.61 B 2.62 A 2.63 B 2.64 B 2.65 B
2.66 D 2.67 C 2.68 D 2.69 C



 
 
 
 


3.1 A vector P is given by 3.5 Divergence of the vector field
 
P  x3 yaˆ x  x 2 y 2aˆ y  x 2 yzaˆ z V ( x, y, z)  ( x cos xy  y)iˆ  ( y cos xy) ˆj
Which of the following statements is [(sin z 2 )  x 2  y 2 ]kˆ is
TRUE? (A) 2 z cos z 2

(A) P is solenoidal but not irrotational. (B) sin xy  2 z cos z 2

(B) P is irrotational but not solenoidal. (C) x sin xy  cos z

(C) P is neither solenoidal nor (D) None of these.
irrotational. 3.6 Divergence of the three-dimensional
 
(D) P is both solenoidal and irrotational. radial vector field r is
3.2 The smaller angle (in degrees) between 1
(A) 3 (B)
the planes x + y + z = 1 and 2x – y + 2z r
= 0 is ______. (C) iˆ  ˆj  kˆ (D) 3(iˆ  ˆj  kˆ)
3.3 The line integral  V dr of the vector
3.7 The two vectors [1, 1, 1] and 1, a, a 2 
function V (r )  2 xyziˆ  x zjˆ  x ykˆ
2 2

 1 3
from the origin to the point P(1,1,1) where a     j  , are
 2 2 
(A) is 1.
(B) is zero. (A) orthonormal. (B) orthogonal.
(C) is – 1. (C) parallel. (D) collinear.
(D) cannot be determined without 3.8 The line integral of function F  yziˆ , in
specifying the path. the counter clockwise direction, along
2 2
x y the circle x 2  y 2  1 at z  1 is
3.4 For the scalar field u   , the
2 3 (A) 2  (B) 
magnitude of the gradient at the point (C)  (D) 2
(1,3) is
3.9 Let   ( f V )  x 2 y  y 2 z  z 2 x, where f
13 9 
(A) (B) and V are scalar and vector field
9 2 
respectively. If V  yiˆ  zjˆ  xkˆ, then
9 
(C) 5 (D)
2 V  (f ) is

 
2 | Engineering Mathematics  
(A) x 2 y  y 2 z  z 2 x 3.15 For the function   ax 2 y  y 3 to
(B) 2 xy  2 yz  2 zx represent the velocity potential of an
(C) x  y  z ideal fluid,  2 should be equal to zero.
(D) 0 In that case, the value of ‘a’ has to be
3.10 The relation that must hold for the flow (A) – 1 (B) 1
to be irrotational is (C) – 3 (D) 3
3.16 The maximum value of the directional
u v u v
(A)  0 (B)  derivative of the function
y x x y
  2 x 2  3 y 2  5 z 2 at a point (1, 1, –1)
 2u  2v u v
(C)  0 (D)  is
x 2 y 2 y x
(A) 10 (B) – 4
3.11 The value of the directional derivative
(C) 152 (D) 152
of the function ( x, y, z )  xy 2  yz 2  zx 2
3.17 Divergence of the vector field
at the point (2, – 1, 1) in the direction of
 x 2 ziˆ  xyjˆ  yz 2 kˆ at (1, –1, 1).
the vector p  iˆ  2 ˆj  2kˆ is
(A) 0 (B) 3
(A) 1 (B) 0.95 (C) 5 (D) 6
(C) 0.93 (D) 0.9   
3.12 The curl of the gradient of the scalar
3.18 Find  F dx where F  x 2
y 2iˆ  yjˆ and C

field defined by is the curve y 2  4 x in xy plane from (0,


V  2 x 2 y  3 y 2 z  4 z 2 x is 0) to (4, 4).

(A) 4 xya x  6 yza y  8 zxa z 3.19 F ( x, y )  ( x 2  xy )aˆ x  ( y 2  xy )aˆ y . Its
line integral over the straight line from
(B) 4ax  6a y  8a z
( x, y)  (0,2) to ( x, y )  (2, 0) evaluates
(C) (4 xy  4 z 2 )ax  (2 x 2  6 yz )a y to
 (3 y  8 zx ) az
2 (A) – 8 (B) 4
(C) 8 (D) 0
(D) 0
3.20 The divergence of vector
3.13 The temperature field in a body varies 
according to the equation T ( x, y)  r  xiˆ  yjˆ  zkˆ is

x3  4 xy. The direction of fastest (A) iˆ  ˆj  kˆ (B) 3


variation in temperature at the point (1, (C) 0 (D) 1
0) is given by 3.21 The value and the integral
 2 xy dx  2 x y dy  dz along a path
2 2
(A) 3iˆ  8 ˆj (B) iˆ
joining. The origin (0, 0, 0) and the
(C) 0.6iˆ  0.8 ˆj (D) 0.5iˆ  0.866 ˆj
point (1, 1, 1).
3.14 The magnitude of the gradient of the 9 1
function f  xyz 3 at (1, 0, 2) is (A) (B)
4 2
(A) 0 (B) 3 1 1
(C) (D)
(C) 8 (D)  3 4
 
Vector Calculus | 3 
3.22 Stoke’s theorem connects 3.29 For the spherical surface
(A) a line integral and a surface integral. x 2  y 2  z 2  1,
(B) a surface integral and a volume the unit outward normal vector at the
integral.
 1 1 
(C) a line integral and a volume integral. point  , ,0  is given by
 2 2 
(D) gradient of a function and its surface
1  1 
integral. (A) i j
υρ υυρ 2 2
3.23 Evaluate ∇ F dx
υρ 1  1 
F  2 y 2 iˆ  3 x 2 ˆj  (2 x  z ) kˆ (B) i j
2 2
Whose vertices (0, 0, 0) (2, 0, 0) (2, 2, 0)
(C) k
32 40
(A) (B) 1  1  1
3 3 (D) i j k
80 10 3 3 3
(C) (D)
3 3 3.30 A vector field
3.24 The divergence of the vector field ( xjˆ  yiˆ) for x 2  y 2  r02
 
( x  y ) iˆ  ( y  x ) ˆj  ( x  y  z ) kˆ is F (r )   r02 ( xjˆ  yiˆ)
 for x 2  y 2  r02
(A) 0 (B) 1  x y
2 2

(C) 2 (D) 3 (A) Find the curl of this field for both of
3.25 The line integral of the vector field region.
F  5 xziˆ  (3 x 2  2 y ) ˆj  x 2 zkˆ along a (B) Find the line integral  A1 dr along

path from (0, 0, 0) to (1, 1, 1)
the semicircle path of radius 2r0 .
parameterized by ( t , t 2 , t ) is ______. ρ
3.31 Surface integral   F Nds ˆ
3.26 Given a vector field
 ρ
F  y 2 xaˆ x  yzaˆ y  x 2 aˆ z , F  18 ziˆ  12 ˆj  3 ykˆ and S is the part of
 
the line integral  F  dl evaluated along surface of the place 2 x  3 y  6 z  12

a segment on the x-axis from x = 1 to x located in XY plane


= 2 is (A) 24 (B) 25
(A) 2.33 (B) 0 (C) 50 (D) 21
(C) 1.33 (D) 7 3.32 Find the surface integral
υρ
3.27 Let C be the boundary of the region   F Nˆ ds
y  x 2 , y  x  2 and x  0 . The value υρ
Where F  yz iˆ  zx ˆj  xy kˆ and S is
 ( xy  y ) dx  x 2 dy .
2
of the integral the part of the surface x 2  y 2  z 2  a 2
3.28 The divergence of the vector field which lies in the XY plane
3 xziˆ  2 xyjˆ  yz 2 kˆ at a point (1, 1, 1) is 3 3
(A) a 2 (B) a 2
equal to 8 8
(A) 7 (B) 4 3 3
(C) a 2 (D) a 4
(C) 3 (D) 0 4 8

 
4 | Engineering Mathematics  
υρ
3.33 Evaluate   FNˆ ds 3.37 Consider a closed surface S surrounding

υρ a volume V. If r is the position vector
F  ( x  y )iˆ  2 xjˆ  2 yxkˆ and S is the of a point inside S, with n̂ the unit
part of the surface x  2 y  3z  6 in the normal on S, the value of the integral

first octant
 5r  nˆ ds is
(A) 0 (B) 2 S

(C) 15 (D) 1 (A) 3 V (B) 5 V


(C) 10 V (D) 15 V
3.34 The following surface integral is to be 
evaluated over a sphere for the given 3.38 The direction of vector A is radially

steady velocity vector field outward from the origin, with A  Kr n ,

F  xiˆ  yjˆ  zkˆ defined with respect to where r 2  x2  y2  z 2 and K is
a Cartesian coordinate system having constant.
iˆ, ˆj and k̂ as unit base vectors. 
The value of n for which   A  0 is
1  (A)  2 (B) 2
S 4 ( F  n) dA (C) 1 (D) 0

Where S is the sphere, x 2  y 2  z 2  1 3.39 Given F  zaˆ x  xaˆ y  yaˆ z . If S
and n is the outward unit normal vector represents the portion of the sphere
to the sphere. The value of the surface x2  y 2  z 2  1 for z  0, then
integral is  
(A)  (B) 2
 
   F d s is ________.
s

3 3.40 The unit normal to the surface at the pt


(C) (D) 4
4  a b c 
 , ,  on the surface of
3.35 Curl of vector  3 3 3

F  x 2 z 2iˆ  2 xy 2 zjˆ  2 y 2 z 3kˆ is ellipsoid.
3.41 Curl of vector
(A) (4 yz 3  2 xy 2 )iˆ  2 x 2 zjˆ  2 y 2 zkˆ
V ( x, y , z )  2 x 2iˆ  3 z 2 ˆj  y 3kˆ at
(B) (4 yz 3  2 xy 2 )iˆ  2 x 2 zjˆ  2 y 2 zkˆ
x  y  z  1 is
(C) 2 xz iˆ  4 xyzjˆ  6 y 2 z 2 kˆ
2
(A) 3iˆ (B) 3iˆ
(D) 2 xz iˆ  4 xyzjˆ  6 y z kˆ
2 2 2
(C) 3iˆ  4 ˆj (D) 3iˆ  6kˆ
3.36 f  x 2  y 2  2 z at P(1, 2, 3) . Find the 3.42 The surface integral
directional derivative in the direction of 1
line PQ where Q(5, 0, 4)  s  (9 xiˆ  3 yjˆ) n dS over the sphere
8 given by x 2  y 2  z 2  9 is ________.
(A) 0 (B)
21 3.43 A scalar potential  has the following
16 18 gradient :
(C) (D)
21 21   yz iˆ  xz ˆj  xy kˆ

 
Vector Calculus | 5 

Consider the integral 
C
 dr on the 3.48 The surface integral  F  n dS
S
over the

curve r  xiˆ  yjˆ  zkˆ. surface S of the sphere x 2  y 2  z 2  9 ,

The curve C is parameterized as where F  ( x  y )iˆ  ( x  z ) ˆj  ( y  z ) kˆ
follows: and n is the unit outward surface
 xt normal, yields ________.
 3.49 The value of the line integral
 y  t and 1  t  3
2

 z  3t 2
  F  r ' ds,
C
where C is a circle of

The value of the integral is ________. 4


radius units is ________.
3.44 The directional derivative of 
xy Here, F ( x, y )  y iˆ  2 x ˆj and r ' is the
f ( x, y )  ( x  y)
2 unit tangent vector on the curve C at an
at (1, 1) in the direction of the unit arc length s from a reference point on
 the curve. iˆ and ĵ are the basis vectors
vector at an angle of with y-axis, is
4 in the x-y Cartesian reference. In
given by________. evaluating the line integral, the curve
3.45 The directional derivative of has to be traversed in the counter-
clockwise direction.
f ( x, y )  2 x  3 y  z at point P (2, 1,
2 2 2

3.50 Consider the two-dimensional velocity


3) in the direction of the vector field given by
a  iˆ  2kˆ is 
V  (5  a1x  b1 y )i  (4  a2 x  b2 y ) j ,
4 4 where a1 , b1 , a2 and b2 are constants.
(A) (B)
5 5
Which one of the following conditions
5  5 needs to be satisfied for the flow to be
(C) (D)
4 4 incompressible?
3.46 Find the directional derivative (A) a1  b1  0 (B) a1  b2  0

f  xy 2  yz 2  zx 2 (C) a2  b2  0 (D) a2  b1  0

Along the tangent to the ensure x  t , 3.51 For the vector V  2 yziˆ  3 xzjˆ  4 xykˆ ,

z  t 3 at (1, 1, 1) y  t 2 . the value of   (  V ) is _________.
 
18 3.52 The value of integral  r  n ds Over
(A) 0 (B) s
12 the closed surface ‘S’ bounding the

28 18 volume, where r  xiˆ  yjˆ  zkˆ is the
(C) (D) 
14 14 position vector and n is normal to the
surface S, is
3.47 The divergence of the vector  yiˆ  xjˆ is
(A) V (B) 2V
________. (C) 3V (D) 4V

 
6 | Engineering Mathematics  
υρ υυρ
3.53 The divergence of the vector field ∇ F dx from (0, 0) to (0, 1) along the

 
u  e x cos yiˆ  sin yjˆ is
path x  t 2 , y  t 3
(A) 0 82 63
(A) (B)
(B) e cos y  e sin y
x x
60 60
(C) 2e x cos y 12
(C) 4 (D)
68
(D) 2e x sin y
3.58 A velocity vector is given as
 
3.54 For a position vector r  xiˆ  yjˆ  zkˆ the V  5 xyiˆ  2 y 2 ˆj  3 yz 2 kˆ
normal of the vector can be defined as
The divergence of this velocity vector at

r  x 2  y 2  z 2 . Given a function (1, 1, 1) is
 (A) 9 (B) 10
  ln r , its gradient  is
 (C) 14 (D) 15
 r
(A) r (B)  3.59 A velocity vector is given as
r 
  V  5 xyiˆ  2 y 2 ˆj  3 yz 2 kˆ
r r
(C)   (D)  3 The divergence of this velocity vector at
r r r
(1, 1, 1) is
3.55 The directional derivative of the scalar (A) 9 (B) 10
function f ( x, y, z )  x 2  2 y 2  z at the (C) 14 (D) 15
point P = (1, 1, 2) in the direction of the 3.60 The directional derivative of

vector a  3iˆ  4 ˆj is f ( x, y, z )  2 x  3 y  z at the point
2 2 2

(A) – 4 (B) – 2 P (2, 1, 3) in the direction of the vector



(C) – 1 (D) 1 a  iˆ  2kˆ is,
3.56 The derivative of f (x, y) at point (1, 2) (A) – 2.785 (B) – 2.145
in the direction of vector iˆ  ˆj is 2 2 (C) – 1.789 (D) 1.000
and in the direction of the vector 2 ˆj is 3.61 A particle moves along a curve whose
 3. Then the derivative of f ( x, y) in parametric equations are :

direction iˆ  2 ˆj is x  t 3  2t , y  3e2t and z = 2 sin (5t)


where x, y and z show variation of the
3 7
(A) 2 2  (B) distance covered by the particle (in cm)
2 5
with time t (in s). The magnetic of the
3 1
(C) 2 2  (D) acceleration of the particle (in cm/s 2 ) at
2 5
υρ t = 0 is ______.
3.57 Let F  x iˆ  ( x  y 2 ) ˆj be a vector 3.62 The directional derivative of the field
field for all x, y with x  0 and u ( x, y, z )  x 2  3 yz in the direction of
ρ
r  x iˆ  y ˆj . Then the value of line the vector (iˆ  ˆj  2kˆ) at point (2, 1, 4)
integral is________.

 
Vector Calculus | 7 
3.63 The divergence of the vector field (A) ( x, y, z )

V  x 2iˆ  2 y 3 ˆj  z 4 kˆ at x  1, y  2,  1 1 1 
(B)  , , 
z  3 is _______.  3 3 3
3.64 The value (up to two decimal places) of  x y z 
line integral (C)  , , 
      3 3 3

c
F ( r )  dr , for F ( r )  x 2iˆ  y 2 ˆj along  x y z 
(D)  , , 
C which is a straight line joining (0, 0)  2 2 2
to (1, 1) is ______. 3.70 For a vector E, which one of the
  following statements is NOT TRUE?
3.65 If a and b are two arbitrary vectors
with magnitudes a and b, respectively, (A) If   E  0, E is called solenoidal.
 2 (B) If   E  0, E is called
a  b will be equal to
conservative.
   
(A) a 2b 2  ( a  b ) 2 (B) ab  a  b (C) If   E  0 , E is called irrotational.
    (D) If   E  0 , E is called irrotational.
(C) a 2b 2  ( a  b ) 2 (D) ab  a  b 
3.71 A vector is defined as f  yiˆ  xjˆ  zkˆ ,
3.66 For a scalar function
f ( x, y , z )  x 2  3 y 2  2 z 2 , where iˆ, ˆj and k̂ are unit vectors in
the directional derivative at the point Cartesian (x, y, z) co-ordinate system.
P(1, 2, 1) in the direction of a vector The surface integral  f  ds over the
iˆ  ˆj  2kˆ is closed surface S of a cube with vertices
having the following co-ordinates :
(A) – 18 (B) 3 6
(0,0,0), (1,0,0), (0,1,0) (0,0,1) (1,0,1)
(C) 3 6 (D) 18 (1,1,1), (0,1,1), (1,1,0) is ______.

3.67 For a scalar function 3.72 Vectors x  5iˆ  2 ˆj  kˆ
1 and
f ( x, y , z )  x  3 y  2 z
2 2 2 
x2  4iˆ  ˆj  2kˆ are transformed into
the gradient at the point P(1, 2, 1) is    
y1  Ax1 and y2  Ax2 using an
(A) 2iˆ  6 ˆj  4kˆ (B) 2iˆ  12 ˆj  4kˆ orthogonal matrix
(C) 2iˆ  12 ˆj  4kˆ (D) 56  2 1 2
1
A   2 2 1 
3.68 The inner (dot) product of two vectors 3
   1 2  2 
P and Q is zero. The angle (degrees) 
The cosine of the angle between y1 and
between the two vectors is

(A) 0 (B) 5 y2 is
(C) 90 (D) 120 8 3
(A) (B)
3.69 A sphere of unit radius is centered at the 70 2
origin. The unit normal vector at a point 1 1
( x , y , z ) on the surface of the sphere is, (C) (D)
70 2

 
8 | Engineering Mathematics  
3.73 The magnitude of the directional 3.78 The angle between two vectors
derivative of the function X 1  [2 6 14]T and
f ( x, y )  x 2  3 y 2 in a direction normal
X 2  [12 8 16]T in radian is _____.
to the circle x  y  2 , at the point (1,
2 2

3.79 The vector that is NOT perpendicular to


1), is
the vectors (iˆ  ˆj  kˆ) and (iˆ  2 ˆj  3kˆ) is
(A) 4 2 (B) 5 2
(A) (iˆ  2 ˆj  kˆ) (B) (iˆ  2 ˆj  kˆ)
(C) 7 2 (D) 9 2
3.74 Which of the following holds for any (C) (0iˆ  0 ˆj  0kˆ) (D) (4iˆ  3 ˆj  5kˆ)
non-zero vector a, 3.80 The directional derivative of
(A)   a  0 (B)   a  0 f ( x, y , z )  x  y  z at the point (1,
2 2 2

(C)   (  a)  0 (D) (  a )  0


1, 1) in the direction iˆ  kˆ is
3.75 A fluid element has a velocity
(A) 0 (B) 1
V   y xiˆ  2 yx 2 ˆj . The motion at
2

(C) 2 (D) 2 2
 1 
( x, y )   ,1 is 3.81 If
 2  υρ
A  (5 x  2 y )iˆ  (3 y  z ) ˆj  (2 x  az ) kˆ
(A) rotational and incompressible.
solenoidal
(B) rotational and compressible.
(A) a  8 (B) a  2
(C) irrotational and compressible.
(C) a  4 (D) a  1
(D) irrotational and incompressible.  
3.76 Determine the following integral, 3.82 If r  xaˆ x  yaˆ y  zaˆ z and r  r , then
I   r dS where r is the position vector div  r 2 (ln r )   _______.
S
3.83 Consider the integral
field (r  xiˆ  yjˆ  zkˆ) and S is the
surface sphere of radius R.   2 xiˆ  2 yjˆ  5z kˆ   nˆ ds
3 3 over the surface of a sphere of radius
(A) 4R 2 (B) R
4  3 with center at origin, and surface
(C) R 3 (D) 4R 3 unit normal n̂ pointing away from the
 origin. Using the Gauss divergence
3.77 Given F   x 2  2 y  iˆ  4 yzjˆ  4 xz 2 kˆ,
theorem, the value of this integral is
  (A) 180  (B) 0
the value of the line integral  .dl
F
c (C) 90  (D) 180 
along the straight line c from (0,0,0) to 3.84 The direction of the largest increase of
(1,1,1) is
the function xy 3  x 2 at the point (1, 1)
3
(A) (B) 0 is
16
(A) 3iˆ  ˆj (B) iˆ  3 ˆj
5
(C) (D) 1
12 (C)  iˆ  3 ˆj (D) iˆ  3 ˆj

 
Vector Calculus | 9 
3.85 The directional derivative of (C)  2 x 2 y 2  3 x 2  iˆ   y 3  3 xy 2  kˆ
1 2
f  x  y2 (D)  3 xy 2  x 3  iˆ   y 3  3 x 2 y  ˆj
2

at (1, 1) in the direction of b  iˆ  ˆj is 3.90 A scalar function in the xy-plane is
1 given by ( x, y )  x 2  y 2 . If iˆ and ĵ
(A) 0 (B)
2 are unit vectors in the x and y directions,
(C) 2 (D) 2 the direction of maximum increase in
3.86 The unit normal vector to the surface of the value of  at (1, 1) is along
the sphere x 2  y 2  z 2  1 at the point (A)  2iˆ  2 ˆj (B) 2iˆ  2 ˆj
 1 1 
 , 0,  is (C)  2iˆ  2 ˆj (D) 2iˆ  2 ˆj
 2 2
( iˆ, ˆj, kˆ are unit normal vectors in the 3.91 A vector u   2 yiˆ  2 xjˆ , where iˆ and
cartesian coordinate system) ĵ are unit vectors in x and y directions,
1 ˆ 1 ˆ respectively. Evaluate the line integral
(A) i j
2 2 I  u dr where, C is a closed loop
1 ˆ 1 ˆ C
(B) i k
2 2 formed by connecting points (1, 1), (3,
1 ˆ 1 ˆ 1), (3, 2) and (1, 2) in that order. The
(C) j k value of I is _______.
2 2
1 ˆ 1 ˆ 1 ˆ 3.92 Let iˆ and ĵ be the unit vectors in the x
(D) i j k
3 3 3 and y directions, respectively. For the
3.87 The value of the surface integral function F ( x, y )  x 3  y 2 the gradient
of the function, i.e.,  F is given by
  ( xiˆ  yjˆ)  nˆ dA evaluated over the
S
(A) 3x 2iˆ  2 yjˆ (B) 6x 2 y
surface of a cube having sides of length
a is (C) 3x 2iˆ  2 yjˆ (D) 2 yiˆ  3x 2 ˆj
(A) 0 (B) a3
3.93 ® (( x
2
 yz ) i  2 x 2 y ˆj  2kˆ ) nˆ ds over S
3 3
(C) 2a (D) 3a
denotes the surface integral
3.88 Gradient of a scalar variable is always
0 xa
(A) a vector (B) a scalar
(C) a dot product (D) zero 0 yb
 
3.89 If u  yiˆ  xyjˆ and v  x 2iˆ  xy 2 ˆj then 0 zc
 
curl  u  v  is abc a 3bc
(A) (B)
3 3
(A)  2 xy 2  iˆ   x  y 2  ˆj
abc abc 3
(B)  xy  x  iˆ   y  3 xy  ˆj
2 (C) (D)
8 3

 
10 | Engineering Mathematics  

3.94 If r is the position vector of any point Where S is the sphere, x 2  y 2  z 2  1
on a closed surface S that encloses the and n is the outward unit normal vector
 
volume V then   (r  ds ) is equal to to the sphere. The value of the surface
s
integral is
1
(A) V (B) 1 V (A)  (B) 2
2
3
(C) 2 V (D) 3 V (C) (D) 4
4
3.95 If F  axiˆ  byjˆ  czkˆ where a, b, c are
3.99 Divergence of the curl of a twice
constants and S is the surface of Umt differentiable continuous vector function
sphere find is
υρ
F Nˆ ds (A) unity (B) infinity
abc (C) zero (D) a unit vector
(A) (a  b  c) (B)  
3 3.100 For two non-zero vectors A and B , if
4    
(C) 0 (D) (a  b  c) A  B is perpendicular to A  B , then
3 
(A) the magnitude of A is twice the
3.96 If T ( x, y , z )  x  y  2 z defines the
2 2 2

magnitude of B .
temperature at any location ( x, y , z ) , 
then the magnitude of the temperature (B) the magnitude of A is half the

gradient at point P(1,1,1) is magnitude of B .
 
(A) 2 6 (B) 4 (C) A and B cannot be orthogonal.
 
(C) 24 (D) 6 (D) the magnitudes of A and B are
P2 equal.
3.97 The line integral  P1
( y dx  x dy) from
P1 ( x1 , y1 ) to P2 ( x2 , y2 ) along the semi-
circle PP
1 2 shown in the figure is

(A) x2 y2  x1 y1
(B) ( y22  y12 )  ( x22  x12 )
(C) ( x2  x1 )( y2  y1 )
(D) ( y2  y1 )2  ( x2  x1 )2
3.98 The following surface integral is to be
evaluated over a sphere for the given
steady velocity vector field
F  xiˆ  yjˆ  zkˆ defined with respect to
a Cartesian coordinate system having
iˆ, ˆj and k̂ as unit base vectors.
1
 4 ( F  n) dA
S

 
Vector Calculus | 11 

Answer Key : Vector Calculus


3.1 A 3.2 54.73 3.3 A 3.4 C 3.5 A
3.6 A 3.7 B 3.8 B 3.9 A 3.10 A
3.11 A 3.12 D 3.13 C 3.14 C 3.15 D
3.16 C 3.17 C 3.18 264 3.19 D 3.20 B
3.21 A 3.22 A 3.23 B 3.24 D 3.25 4.4167
3.26 B 3.27 * 3.28 C 3.29 A 3.30 ***
3.31 * 3.32 D 3.33 C 3.34 A 3.35 A
3.36 D 3.37 D 3.38 A 3.39 3.14 3.40 ***
3.41 A 3.42 216 3.43 726 3.44 3 3.45 B
3.46 D 3.47 0 3.48 226.19 3.49 16 3.50 B
3.51 0 3.52 C 3.53 C 3.54 C 3.55 B
3.56 B 3.57 A 3.58 D 3.59 D 3.60 C
3.61 12 3.62 5.715 3.63 134 3.64 0.67 3.65 A
3.66 * 3.67 B 3.68 C 3.69 A 3.70 D
3.71 1 3.72 A 3.73 A 3.74 C 3.75 A
3.76 D 3.77 D 3.78 0.723 3.79 D 3.80 A
3.81 A 3.82 3 3.83 D 3.84 C 3.85 A
3.86 B 3.87 C 3.88 A 3.89 D 3.90 B
3.91 8 3.92 C 3.93 B 3.94 * 3.95 D
3.96 * 3.97 * 3.98 * 3.99 * 3.100 *

Note : ***
  bciˆ  cajˆ  abkˆ 
3.30 Region I  (  F ) region  2kˆ 3.40  2 2 
 b c c a a b
2 2 2 2


(A) (  F ) region 2  0 (B) r02



 
 
 
 

2 0 0 1 1 0  2
0 1 0 0   1 
4.1 A . (A)  0 0
2 0 3 0  3 
   2 0 5 
 1 0 0 4 
The sum of the Eigen values of the 5 0 2
matrix A is  1 
(B) 0  0
(A) 10 (B) –10  3 
(C) 24 (D) 22 2 0 1 

4.2 Express the given matrix,
1 1
2 1 5  5 0 2
A   4 8 13 
1

 6 27 31 (C)  0 0
 3 
 
as a product of triangular matrices L and 1 0 1
U where the diagonal elements of the  2 
lower triangular matrix L are unity and  1 1
U is an upper triangular matrix.  5 0  2
 
1  1
(D) 0 0 
4.3 The vector  2  is an Eigen vector of  3 
   
 1  1 0 1 
 2 
 2 2  3
4.5 A set of linear equation is represented
A   2 1  6  .
by the matrix equation AX  B. The
  1  2 0  necessary condition for the existence of
One of the Eigen value of A is a solution for this system is
(A) 1 (B) 2 (A) A must be invertible.
(C) 5 (C) –1 (B) B must be linearly dependent on the
5 0 2 columns of A.
4.4 If A   0 3 0  then A1  (C) B must be linearly independent of
  the columns of A.
 2 0 1 
(D) None.

 
2 | Engineering Mathematics  
4.6 The determinant of the matrix l  3    3
 1 0 0 0 (A)   2  (B)  2 
100 1    
 0 0   1   1
is
100 200 1 0
   1  2
100 200 300 1
(C)   2  (D)  5 
   
(A) 100 (B) 200  3   0 
(C) 1 (D) 300
X   x1 x2 ..... xn  is an n-tuple non-
T
4.7 The Eigen values of the system 4.11
0 1 0 0  zero vector.
0 0 1 0 
represented by X    are The n  n matrix V  XX T
0 0 0 1 
  (A) has rank zero. (B) has rank 1.
0 0 0 1 
(C) is orthogonal. (D) has rank n.
(A) 0, 0, 0, 0 (B) 1, 1, 1, 1
4.12 Let x and y be two vectors in a 3
(C) 0, 0, 0, –1 (D) 1, 0, 0, 0
dimensional space and  x, y  denote
 1 0 1
their dot product. Then the determinant
4.8 If R   2 1 1 then the top row of
 
 2 3 2    x, x   x, y  
det  
R 1 is   y, x   y, y  

(A) 5 6 4 (B) 5  3 1 (A) is zero when x and y are linearly


independent.
(C)  2 0 1 (D)  2 1 0
(B) is positive when x and y are linearly
4.9 In the matrix equation PX  Q which of independent.
the following is a necessary condition (C) is non-zero for all non-zero x and y.
for the existence of at least one solution
(D) is zero only when either x or y is
for the unknown vector X
zero.
(A) Augmented matrix  P : Q must
4.13 The characteristic equation of a (3  3)
have the same rank as matrix P.
matrix P is defined as,
(B) Vector Q must have only non-zero
elements.  ()  I  P  3  2  2  1  0.
(C) Matrix P must be singular. If I denotes identity matrix, then the
(D) Matrix P must be square. inverse of matrix P will be
3  2 2 (A) P 2  P  2 I
4.10 For the matrix P   0  2 1  , one of
  (B) P 2  P  I
 0 0 1 
(C)  ( P 2  P  I )
the Eigen values is equal to – 2. Which
of the following is an Eigen vector? (D)  ( P 2  P  2 I )

 
Linear Algebra | 3 
4.14 Let P be a 2  2 real orthogonal matrix 1 0  1 1 
 (A)   and  
and x is a real vector  x1 x2  with
T
4 1 0  2 
 2 0  1 1
length x  ( x12  x22 )1/2 . Then which one
(B)   and  
of the following statements is correct? 4 1 0 1
  1 0 2 1 
(A) Px  x where at least one vector (C)  and
 0 1
 
satisfies Px  x .
4 1  
   2 0  1 0.5
(B) Px  x for all vectors x . (D)   and  
 4  3 0 1 
 
(C) Px  x where at least one vector 4.18 The trace and determinant of a 2  2
  matrix are known to be –2 and –35
satisfies Px  x .
respectively. It’s Eigen values are
(D) No relationship can be established (A) –30 and –5 (B) –37 and –1
  (C) –7 and 5 (D) 17.5 and –2
between x and Px .
4.19 If the rank of a (5  6) matrix Q is 4,
4.15 A is a m  n full rank matrix with m  n
then which one of the following
and I is an identity matrix. Let matrix
statements is correct?
A  ( AT A) 1 AT . Then which one of the (A) Q will have four linearly independent
following statements is FALSE? rows and four linearly independent
(A) AA A  A (B) ( AA ) 2  AA columns.
(B) Q will have four linearly independent
(C) A A  I (D) AA A  A
rows and five linearly independent
4.16 For the set of equations, columns.
x1  2 x2  x3  4 x4  2 (C) QQT will be invertible.
and 3 x1  6 x2  3 x3  12 x4  6 . (D) QT Q will be invertible.
Which of the following statements is 4.20 Given that,
true?   5  3 1 0
A  and I   
(A) Only the trivial solution x1  x2  2 0 0 1 
x3  x4  0 exists. The value of A3 is
(B) There are no solution. (A) 15 A  12 I (B) 19 A  30 I
(C) A unique non-trivial solution exists. (C) 17 A  15 I (D) 17 A  21 I
(D) Multiple non-trivial solutions exist.  2  2  x1  0
4.21 The equation        has
2 1  1 1   x2  0
4.17 The matrix  A    is decomposed
4 1 (A) no solution.
into a product of a lower triangular  x1  0
(B) only one solution      .
matrix [L] and an upper triangular  x2  0
matrix [U]. The properly decomposed (C) non-zero unique solution.
[L] and [U] matrices respectively are (D) multiple solution.

 
4 | Engineering Mathematics  
4.22 A matrix has Eigen values –1 and –2. 4.26 Two matrices A and B are given below,
The corresponding Eigen vectors are  p q  p2  q2 pr  qs 
1 1 A  ; B   .
 1 and   2  respectively. r s  pr  qs r 2  s2 
   
If the rank of matrix A is N, then the
The matrix is
rank of matrix B is
1 1  1 2
(A)   (B)   (A) N /2 (B) N 1
1  2   2  4
(C) N (D) 2N
1 0  0 1
(C)   (D)   4.27 If the sum of the diagonal elements of a
 0  2   2  3 2  2 matrix is –6, then the maximum
4.23 Given a system of equations possible value of determinant of the
x  2 y  2 z  b1 matrix is _______.
5 x  y  3 z  b2 4.28 The maximum value of ‘a’ such that the
which of the following is true regarding  3 0  2
its solutions? matrix  1  1 0  has three linearly
 
(A) The system has a unique solution for  0 a  2 
any given b1 and b2 .
independent real Eigen vectors is
(B) The system will have infinitely
2 1
many solution for any given b1 and (A) (B)
3 3 3 3
b2 .
1 2 3 1 3
(C) Whether or not a solution exists (C) (D)
depends on the given b1 and b2 . 3 3 3 3
(D) The system would have no solution 3 1
4.29 Let P    . Consider the set S of all
for any values of b1 and b2 . 1 3
4.24 A system matrix is given as follows  x
vectors   such that a 2  b 2  1 where
0 1  1  y
A    6  11 6  .

a  x
  6  11 5     P   . Then S is
b  y
The absolute value of the ratio of the
maximum Eigen value to the minimum (A) a circle of radius 10 .
Eigen value is _______. 1
(B) a circle of radius .
4.25 Which one of the following statements 10
is true for all real symmetric matrices?
 1
(A) All the Eigen values are real. (C) an ellipse with major axis along  
(B) All the Eigen values are positive.  1
(C) All the Eigen values are distinct.  1
(D) an ellipse with minor axis along  
(D) Sum of all the Eigen values is zero.  1
 
Linear Algebra | 5 
4.30 A 3 3 matrix P is such that, P3  P . 4.34 The Eigen value of the matrix given
Then the Eigen values of P are below are
(A) 1, 1,  1 0 1 0 
0 0 1 
(B) 1, 0.5  j 0.866, 0.5  j 0.866 
 0  3  4 
(C) 1,  0.5  j 0.866,  0.5  j 0.866
(D) 0, 1,  1 (A) 0,  1,  3 (B) 0,  2,  3
(C) 0, 2, 3 (D) 0,1, 3
4.31 Let the Eigen values of a 2  2 matrix A
be 1, – 2 with Eigen vectors x1 and x2 4.35 Consider a non-singular 2  2 square
matrix A. If trace ( A )  4 and trace
respectively. Then the Eigen values and
Eigen vectors of the matrix A 2  3 A  4 I ( A2 )  5 . The determinant of the matrix
would respectively be A is _______ (up to one decimal place).
(A) 2,14; x1 , x2  1 0 1 
4.36 Let A    1 2 0  and
(B) 2,14; x1  x2 , x1  x2  
 0 0  2 
(C) 2, 0; x1 , x2
B  A3  A 2  4 A  5 I
(D) 2, 0; x1  x2 , x1  x2 where, I is the 3 3 identity matrix. The
4.32 Consider a 3 3 matrix with every determinant of B is _______. (up to one
element being equal to 1. Its only non- decimal place).
zero Eigen values is _______. 1 1 1
3 1 4.37 The Eigen values of 1 1 1 are
 
2 0 2 1 1 1
 
4.33 The matrix A   0  1 0  has three (A) 0, 0, 0 (B) 0, 0, 1
1 3
 0  (C) 0, 0, 3 (D) 1, 1, 1
2 2 4.38 Which one of the following is Eigen
distinct Eigen values and one of its 5 0 0 0
1  0 5 0 0
vector of the matrix  ?
Eigen vectors is  0  . Which one of the 0 0 2 1 
 
 1   
0 0 3 1 
following can be another Eigen vector 1  0 
of A?  2 0 
(A)   (B)  
0   1  0  1 
(A)  0  (B)  0     
     0  0 
  1  0 
1  1
1 1  0   1
(C)   (D)  
(C)  0  (D)   1  0  2
   
  1  1     
 2 1

 
6 | Engineering Mathematics  
4.39 Multiplication of matrices E and F is G. 1 2 
Matrices E and G are 4.42 The Eigen vectors of the matrix  
0 2
 cos   sin  0  1  1 
E   sin  cos  0  and are written in the form   and   .
a  b 
 0 0 1 
What is a  b ?
1 0 0  (A) 0 (B) 1/2
G   0 1 0  . (C) 1 (D) 2
 0 0 1  4.43 If a square matrix A is real and
symmetric, then the Eigen values
What is the matrix F? (A) are always real.
 cos   sin  0  (B) are always real and positive.
(A)  sin  cos  0  (C) are always real and non-negative.
  (D) occur in complex conjugate pairs.
 0 0 1 
4.44 Match the items in columns I and II.
 cos  cos  0  Column I Column II
(B)   cos  sin  0  P. Singular 1. Determinant is
 
 0 0 1  matrix not defined
Q. Non-square 2. Determinant is
 cos  sin  0  matrix always one
(C)   sin  cos  0  R. Real 3. Determinant is
 
 0 0 1  symmetric zero
S. Orthogonal 4. Eigen values are
 sin   cos  0 
matrix always real
(D)  cos  sin  0 
  5. Eigen values are
 0 0 1 
not defined
 3 2 (A) P-3, Q-1, R-4, S-2
4.40 Eigen values of a matrix S    are
 2 3 (B) P-2, Q-3, R-4, S-1
(C) P-3, Q-2, R-5, S-4
5 and 1. What are the Eigen values of
(D) P-3, Q-4, R-2, S-1
the matrix S 2  SS ?
4.45 x  2 y  z  4,
(A) 1 and 25 (B) 6 and 4
2 x  y  2 z  5,
(C) 5 and 1 (D) 2 and 10 x  y  z 1
1 2 4 The system of algebraic equations given
4.41 The matrix 3 0 6  has one Eigen above has

1 1 p  (A) a unique solution of x  1, y  1 and
value equal to 3. The sum of the other z  1.
(B) only the two solutions of x  1, y  1,
two Eigen values is
z  1 and x  2, y  1, z  0 .
(A) p (B) p  1
(C) infinite number of solutions.
(C) p  2 (D) p  3 (D) no feasible solution.

 
Linear Algebra | 7 
4.46 Choose the correct set of functions, 1  i 4  3i 
(B)
which are linearly dependent. 25 4  3i  i 

(A) sin x, sin 2 x and cos 2 x
1  4  3i  i 
(B) cos x , sin x and tan x (C)
24  i 4  3i 
(C) cos 2 x, sin 2 x and cos 2 x
(D) cos 2 x , sin x and cos x 1  4  3i  i 
(D)
4.47 Given that the determinant of the matrix 25  i 4  3i 
 1 3 0 4.52 The condition for which the Eigen values
 2 6 4  is –12, the determinant of 2 1 
  of the matrix A    are positive, is
  1 0 2  1 k 
 2 6 0 1
(A) k  (B) k   2
the matrix  4 12 8  is 2
 
1
  2 0 4  (C) k  0 (D) k 
2
(A)  96 (B)  24
4.53 A real square matrix A is called skew-
(C) 24 (D) 96
symmetric if
4.48 Consider a 3 3 real symmetric matrix
S such that two of its Eigen values are (A) AT  A (B) AT  A1
a  0, b  0 with respective Eigen (C) AT   A (D) AT  A  A1
 x1   y1  4.54 The product of Eigen values of the
matrix P is
vectors  x2  ,  y2  .
    2 0 1
 x3   y3 
P   4  3 3 
If a  b then x1 y1  x2 y2  x3 y3 equals
 0 2  1
(A) a (B) b
(C) ab (D) 0 (A) – 6 (B) 2
4.49 At least one Eigen value of a singular (C) 6 (D) – 2
matrix is 4.55 Consider the matrix
(A) Positive (B) Zero  1 1 
(C) negative (D) Imaginary  2 0 2
 
4.50 The lowest Eigen value of the 2  2 P 0 1 0 
 4 2  1 1 
matrix   is _______.  0 
1 3  2 2 
4  3i  i  Which one of the following statements
4.51 For a given matrix P   ,
 i 4  3i  about P is incorrect?
(A) Determinant of P is equal to 1.
where i  1, the inverse of matrix P
(B) P is orthogonal.
is
(C) Inverse of P is equal to its transpose.
1  4  3i i  (D) All Eigen values of P are real
(A)
24   i 4  3i 

numbers.

 
8 | Engineering Mathematics  
50 70 3 4
4.56 Consider the matrix A    whose  5
70 80  4.61 For a matrix M   5 , the
Eigen vectors corresponding to Eigen x 3
values 1 and  2 are  5 
transpose of the matrix is equal to the
 70   2  80
x1    and x2   , inverse of the matrix  M    M  . The
T 1
1  50  70 
respectively. The value of x1T x2 is ____. value of x is given by
4 3
1 2 3 (A)  (B) 
If A   0 4 5 the det ( A1 ) is ______
4.57
5 5
  3 4
 0 0 1  (C) (D)
5 5
(correct to two decimal places).
4.58 If A and B are square matrices of size 4.62 The Eigen values of a square symmetric
n  n, then which of the following matrix are always
statement is not true? (A) positive
(A) det ( AB )  det ( A) det ( B ) (B) real and imaginary
(B) det (kA)  k n det ( A) (C) negative
(C) det ( A  B )  det ( A)  det ( B ) (D) real
(D) det ( AT )  1/ det ( A1 ) 4.63 If the determinant of the matrix
4.59 If matrix A is m  n and B is n  p, the 1 3 2 
number of multiplication operations and  0 5  6  is 26 then the determinant
addition operations needed to calculate  
 2 7 8 
the matrix AB, respectively, are
(A) mn 2 p, mpm 2 7 8 
(B) mpn , mp ( n  1) of the matrix  0 5  6  is
 
(C) mpn, mpn  1 3 2 
(D) mn 2 p, (m  p ) n (A) – 26 (B) 26
4.60 Let A be an invertible matrix and (C) 0 (D) 52
suppose that the inverse of 7A is
4.64 If A is a real square matrix then AAT is
1 2 
 4  7 , the matrix A is (A) unsymmetric
 
(B) always symmetric
 2
1 (C) non-singular
7 7 2
(A)   (B)   (D) sometimes symmetric
4 1 4 1
 7 4.65 If A is any n  n matrix and k is a scalar
7 
 4 then kA   A where  is
 1 7  7 4 (A) kn (B) n k
(C)   (D)  
2 1  2 1 k
 7 7  (C) k n (D)
n
 
Linear Algebra | 9 
4.66 The number of terms in the expansion of 4.71 Consider the following system of
general determinant of order n is equations in three real variable x1 , x2
(A) n 2 (B) n !
and x3 :
(C) n (D) (n  1)2
2 1 1 2 x1  x2  3 x3  1
4.67 The equation 1 1  1  0 represents 3 x1  2 x2  5 x3  2
y x2 x  x1  4 x2  x3  3
a parabola passing through the points.
This system of equation has
(A) (0, 1), (0, 2), (0, –1)
(B) (0, 0), (–1, 1), (1, 2) (A) no solution
(C) (1, 1), (0, 0), (2, 2) (B) a unique solution
(D) (1, 2), (2, 1), (0, 0) (C) more than one but a finite number of
4.68 The determinant of the following matrix solutions.
5 3 2  (D) an infinite number of solution.
1 2 6 
   2  2 3
3 5 10 
4.72 For given matrix A    2  1 6  , one
(A) –76 (B) –28  
 1 2 0 
(C) 28 (D) 72
4.69 Consider the matrices X 43 , Y43 and of the Eigen values is 3. The other two
T Eigen values are
P23 . The order of  P ( X T Y ) 1 PT  will
(A) 2, – 5 (B) 3, – 5
be
(C) 2, 5 (D) 3, 5
(A) 2  2 (B) 3 3
(C) 4  3 (D) 3 4 4.73 The maximum and the maximum Eigen
4.70 Consider the system of equations Ann 1 1 3 
xnt   nt where,  is a scalar. Let  i , values of the matrix 1 5 1  are –2
 
xi be an Eigen pair of an Eigen value 3 1 1 
and its corresponding Eigen vector for and 6 respectively. What is the other
real matrix. Which one of the following Eigen value?
statements is NOT correct?
(A) 5 (B) 3
(A) For a homogeneous n  n system of
line at equations, ( A   I ) x  0 having (C) 1 (D) –1
a nontrivial solution, the rank of 4.74 For what values of  and  the
( A   I ) is less than n. following simultaneous equations have
m
(B) For matrix A , m being a positive an infinite number of solutions?
integer, (im , xim ) will be the Eigen x  y  z  5, x  3 y  3 z  9,
pair for all i. x  2 y  z  
(C) If AT  A1 , then i  1 for all i. (A) 2, 7 (B) 3, 8
(D) If AT  A, then  i is real for all i. (C) 8, 3 (D) 7, 2

 
10 | Engineering Mathematics  
1 2 3 2 1
4.75 The inverse of 2  2 matrix   is
5 7 4.80 Given the matrices J   2 4 2  and
 
 1 2 6 
1  7 2  1 7 2
(A) (B)
3  5 1 3 5 1  1
K   2  , the product K T JK is _____.
1  7  2 1   7  2
(C) (D)   1
3  5 1  3   5 1
4.81 The sum of the Eigen value of the matrix,
1
4.76 The product of matrices ( PQ ) P is  M  is
1 1
(A) P (B) Q  215 650 795
1
(C) P Q P 1
(D) PQP 1
where,  M    655 150 835 
 
4.77 The Eigen values of the matrix  485 355 550 
4 5  (A) 915 (B) 1355
P  are (C) 1640 (D) 2180
 2  5
4.82 The determinant of matrix
(A) – 7 and 8 (B) – 6 and 5
0 1 2 3
(C) 3 and 4 (D) 1 and 2. 1 0 3 0
4.78 The following system of equations   is _______.
2 3 0 1
x  y  z  3, x  2 y  3 z  4,  
3 0 1 2
x  4y  k  6
4.83 Which one of the following statements
Will not have a unique solution for k is TRUE about every n  n matrix with
equal to only real Eigen values?
(A) 0 (B) 5 (A) If the trace of the matrix is positive
(C) 6 (D) 7 and the determinant of the matrix is
negative, at least one of its Eigen
4.79 The inverse of the matrix
values is negative.
3  2i i  (B) If the trace of the matrix is positive,
  i 3  2i  is
  all its Eigen values are positive.
(C) If the determinant of the matrix is
1 3  2i  i 
(A) positive, all its Eigen values are
12  i 3  2i 
positive.
1 3  2i  i  (D) If the product of the trace and
(B)
12  i 3  2i  determinant of the matrix is positive,
all its Eigen values are positive.
1 3  2i  i  4.84 The rank of matrix
(C)
14  i 3  2i  6 0 4 4 
  2 14 8 18  is _______.
1 3  2i  i   
(D)
14  i 3  2i   14  14 0  10 

 
Linear Algebra | 11 
4.85 With reference to the conventional The characteristics equation for these
Cartesian ( x , y ) coordinate system, the simultaneous equation is
vertices of a triangle have the following (A)  2  4   5  0
coordinates :
(B)  2  4   5  0
( x1 , y1 )  (1, 0); ( x2 , y2 )  (2, 2);
(C)  2  4   5  0
( x3 , y3 )  (4,3)
The area of triangle is equal to (D)  2  4   5  0
(A) 3/2 (B) 3/4 4.91 For the given orthogonal matrix Q,
(C) 4/5 (D) 5/2  3 2 6 
4.86 The smallest and largest Eigen values of  7 7 7 
the following matrix are  
6 3 2 
3  2 2 Q  
 7 7 7 
4  4 6  
   2 6

3
 2  3 5   7 7 7 
(A) 1.5 and 2.5 (B) 0.5 and 2.5 The inverse is
(C) 1.0 and 3.0 (D) 1.0 and 2.0
 3 2 6 
4.87 The two Eigen values of the matrix  7 7 7 
2 1   
1 p  have a ratio of 3 : 1 for p  2 . 6 3 2 
  (A)  
 7 7 7 
What is another value of p for which the  
Eigen values have the same ratio of 3 : 1  2 6 3
 
 7 7 7 
(A)  2 (B) 1
7 14  3 2 6
(C) (D)  7 
7
 
7
3 3  
6 3 2
4.88 If the entries in each column of a square (B)   
matrix M add up to 1, then an Eigen  7 7 7
 
value of M is  2 
6 3 
(A) 4 (B) 3  7 7 7 
(C) 2 (D) 1
3 6 2 
4.89 Consider the following linear system 7 
x  2 y  3z  a 7 7 
 
2 3 6 
2 x  3 y  3z  b (C) 
5x  9 y  6z  c
7 7 7 
 
The system is consistent if a , b and c 6 2 3
 
satisfy the equation  7 7 7 
(A) 7a  b  c  0 (B) 3a  b  c  0  3 6 2
(C) 3a  b  c  0 (D) 7a  b  c  0  7 7
 
7
 
4.90 Consider the following simultaneous 2 3 6
(D)    
equations (with c1 and c2 being  7 7 7
constants) :  
 6 
2 3 
3 x1  2 x2  c1 , 4x1  x2  c2  7 7 7 

 
12 | Engineering Mathematics  
4.92 For a given 2  2 matrix A, it is observed 0 0 1 
that 4.96 The matrix P  1 0 0  rotates a
 
1 1 1 1  0 1 0 
A       and A     2   .
1 1   2   2 1
Then matrix A is vector about the axis 1 by an angle of
 
1
 2 1  1 0  1 1 
(A) A     1  2
1 1  0  2   (A) 30 0 (B) 60 0
(C) 90 0 (D) 120 0
1 1  1 0   2 1 
(B) A    0 2 1 1 4.97 A real n  n matrix A   aij  is define
1  2   
as follows :
 1 1  1 0   2 1  aij  i ; if i  j
(C) A     
1  2  0  2 1 1  0; otherwise
0 2 The summation of all n Eigen values of
(D) A   
1 3 A is
4.93 Let A be n  n real matrix such that (n  1) (n  1)
(A) n (B) n
2 2
A2  I and y be n-dimensional vector.
n (n  1) (2n  1)
Then the linear system of equations (C) (D) n 2
6
Ax  y has
 2 2  3
(A) no solution 4.98 The matrix has M   2 1  6 

(B) a unique solution   1  2 0 
(C) more than one but finitely many Eigen values  3,  3, 5 . An Eigen vector
independent solution corresponding to the Eigen value 5 is
(D) infinitely many independent solutions.
1 2  1 . One of the Eigen vectors
T

4.94 Let P  0 be a 3 3 real matrix. There


of the matrix M 3 is
exist linearly independent vectors x and
(A) 1 8  1 (B) 1 2  1
T T

y such that Px  0 and Py  0 . The


T
(D) 1 1  1
T
dimension of the range space of P is (C) 1 3 2  1
(A) 0 (B) 1 4.99 X and Y are non-zero square matrices of
(C) 2 (D) 3 size n  n . If XY  0nn then
4.95 The Eigen values of a 2  2 matrix X are (A) X  0 and Y  0
– 2 and – 3. The Eigen values of the
(B) X  0 and Y  0
matrix ( X  I ) 1 ( X  5I ) are
(A) – 3, – 4 (B) – 1, – 2 (C) X  0 and Y  0

(C) – 1, – 3 (D) – 2, – 4 (D) X  0 and Y  0

 
Linear Algebra | 13 
4.100 One pair of Eigen vectors corresponding 4.106 Given that
to the two Eigen values of the matrix   5  3 1 0 
A  and I   ,
0 1 2 0 0 1 
1 0  is
  the value of A3 is
1  j 0 1 (A) 15 A  12I (B) 19 A  30I
(A)   ,  1 (B)   , 0
 j    1    (C) 17 A  15I (D) 17 A  21I
4.107 The Eigen values of the matrix
 1  0  1   j 
(C)   ,   (D)   ,    1 1 5 
 j  1   j  1 
A  0 5 6  are
4.101 For the matrix A satisfying the equation 0  6 5 
given below, the Eigen values are
(A)  1, 5, 6 (B) 1, 5, 4
1 2 3 1 2 3
(C) 1, 5  j 6 (D) 1, 5, 5
 A  7 8 9    4 5 6 
4.108 If V is a non-zero vector of dimension
 4 5 6   7 8 9 
3  1, then the matrix A  VV T has rank
(A) 1,  j , j (B) 1,1, 0  _______.
(C) 1,1,  1 (D) 1, 0, 0 4.109 The value of the following determinant
4.102 Let A be an n  n matrix with rank r, 1 0 0 0 0
0  r  n . Then AX  0 has p 2 2 0 0 0
independent solutions, where p is 3 5 3 0 0 is
(A) r (B) n 1 4 7 4 0
(C) n  r (D) n  r 5 6 3 1 1
4.103 A straight line of the form y  mx  c (A) 24 (B) 32
passes through the origin and the point (C) 16 (D) 8
( x , y )  (2, 6) . The value of m is _____. 4.110 Let N be a 3 3 matrix with real number
entries. The matrix is such that N 2  0 .
2 1 1 
 The Eigen values of N are
4.104 Consider the matrix A  2 3
 4 
(A) 0, 0, 0 (B) 0, 0, 1
 1 1  2 
(C) 0, 1, 1 (D) 1, 1, 1
whose Eigen values are 1, –1 and 3. The 4.111 Consider the following system of linear
trace of ( A3  3 A2 ) is _______. equations :
4.105 The dimension of the null space of the 3 x  2 ky   2
0 1 1 kx  6 y  2
matrix  1 1 0  is Here x and y are the unknowns and k is a
 
 1 0 1 real constant. The value of k for which
there are infinite number of solutions is
(A) 0 (B) 1 (A) 3 (B) 1
(C) 2 (D) 3 (C) – 3 (D) – 6

 
14 | Engineering Mathematics  
4.112 The matrix A is given by, 4.116 Consider the following set of linear
1 4 algebraic equations
A 
a 2 x1  2 x2  3x3  2
The Eigen values of the matrix A are x2  x3  1
real and non-negative of the condition
2 x2  2 x3  0
1 1 1 1
(A)   a  (B)   a 
16 16 2 2 The system has,
1 1 1 1 (A) a unique solution.
(C)   a  (D)   a 
2 16 16 2 (B) no solution.
4.113 A and B are two 3 3 matrix such that (C) an infinite number of solution.
 2 4 6 (D) only the trivial solution.
A   1 2 1  , B  0 4.117 Which of the following statements are
 0 4 4  TRUE?
and A  B  0 . Then the rank of matrix B P. The Eigen values of a symmetric
is matrix are real.
(A) r  2 (B) r  3 Q. The value of the determinant of an
(C) r  3 (D) r  3 orthogonal matrix can only be +1.
1 2 R. The transpose of a square matrix A
4.114 The Eigen values of matrix A    has the same Eigen values as those
 4 3
of A.
are 5 and –1. Then the Eigen value of
 2 A  3 I [I is a 2  2 identity matrix] S. The inverse of an ‘n  n’ matrix
exists if and only if the rank is less
are
than ‘n’.
(A)  7 and 5 (B) 7 and  5
(A) P and Q only (B) P and R only
7 1 1 1
(C)  and (D) and  (C) Q and R only (D) P and S only
5 5 7 2
4.115 Let, 1   1 and  2  3 be the Eigen 4.118 The following set of three vectors
 1   1 1   x  3
values and V1    and V2    be the  2  ,  6  and  4  is linearly dependent
0  1      
1   x   2 
corresponding Eigen vectors of a real
 when x is equal to
2  2 matrix R . Given that P  V1V2 ,
(A) 0 (B) 1
which one of the following matrices
(C) 2 (D) 3
1
represents P R P ? cos   sin  
4.119 For the matrix A    if
 0 1  0 3  sin  cos  
(A)   (B)  
3 0   1 0 det stands for the determinant and AT is
3 0   1 0 the transpose of A then the value of det
(C)   (D)  
 0 1 0 3 ( AT A) is _______.

 
Linear Algebra | 15 
1  4  1 b b 1
4.120 The matrix   is an inverse of
1  5  4.125 The determinant b 1  b 1
matrix 1 2b 1
5  4  3  4 evaluates to
(A)   (B)  
1  1  2 1  (A) 0 (B) 2b (b  1)
 1  4   4  3
(C)   (D)   (C) 2 (1  b ) (1  2b ) (C) 3b (1  b )
2 3   2 1 
4.126 q1 ,... qm are n-dimensional vectors with
4.121 The value of the following determinant
1 4 9 m  n . This set of vectors is linearly
4 9 16 is dependent. Q is the matrix with q1 ,... qm
9 16 25 as the columns. The rank of Q is
(A) 8 (B) 12 (A) Less than m (B) m
(C) –12 (D) –8 (C) Between m and n (D) n
4.122 If for a matrix, rank equals both the 4.127 The Eigen vector pair of the matrix
number of rows and number of columns, 3 4 
then the matrix is called  4  3 is
 
(A) Non-singular (B) Singular
(C) Transpose (D) Minor 2  1
(A)   and  
2 1 1   2 
4.123 Given matrix L   3 2  and 2  1 
 
 4 5  (B)   and  
1  2 
3 2
M   then L  M is  2  1
0 1  (C)   and  
1  2 
 8 1 6 5
   2  1 
(A) 13 2 (B)  9 8  (C)   and  
   
 22 5  12 13 1 2 
 5 3
1 8  6 2 4.128 For the matrix A    , ONE of the

(C) 2 13  (D)  9 4  1 3 
   
 5 22   0 5  normalized Eigen vectors is given as

4.124 The Eigen values of the matrix M are  1   1 


 2   2 
given as 15, 3 and 0. (A)   (B) 
 8 6 2   3  1 
   
M    6 7  4  , the value of the  2   2
 2  4 3   3   1 
 10   5 
determinant of a matrix is (C)  (D) 
(A) 20 (B) 10  1   2 
   
(C) 0 (D) –10  10   5

 
16 | Engineering Mathematics  
4.129 x  2 y  z  4, 4.132 For an orthogonal matrix Q, the valid
2 x  y  2 z  5, equality is
x  y  z 1 (A) QT  Q 1 (B) Q  Q 1
The system of algebraic equations given (C) QT  Q (D) det (Q )  0
above has 4.133 The dia
(A) A unique solution of x  1, y  1 and gonal elements of a 3-by-3 matrix are 10, 5
z  1. and 0, respectively. If two of its Eigen
(B) Only the two solution of ( x  1, values are –15 each, the third Eigen
y  1, z  1 ) and ( x  2, y  1, z  0 ). value is _______.
(C) Infinite number of solution.
(D) No feasible solution.
4.130 The system of equation given below, has
x  2 y  4z  2
4x  3y  z  5
3x  2 y  3z  1
(A) a unique solution
(B) two solutions
(C) no solution
(D) more than two solutions
4.131 Match the linear transformation matrices
listed in the first column to their
interpretations in the second column :
P. 1 0  1. Stretch in
0 0 the y-axis
 
Q. 0 0 2. Uniform
0 1  stretch in x
 
and y-axis
R. 1 0  3. Projection in
0 3 x- axis
 
S. 4 0 4. Projection in
0 4 y-axis
 
(A) P-1, Q-2, R-3, S-4
(B) P-2, Q-3, R-4, S-1
(C) P-3, Q-4, R-1, S-2
(D) P-4, Q-1, R-2, S-3

 
Linear Algebra | 17 

Answer Key : Linear Algebra


4.1 A 4.2 * 4.3 C 4.4 A 4.5 B
4.6 C 4.7 D 4.8 B 4.9 A 4.10 D
4.11 B 4.12 B 4.13 D 4.14 B 4.15 D
4.16 D 4.17 D 4.18 C 4.19 A 4.20 B
4.21 D 4.22 D 4.23 B 4.24 0.33 4.25 A
4.26 C 4.27 9 4.28 B 4.29 D 4.30 D
4.31 A 4.32 3.0 4.33 C 4.34 A 4.35 5.5
4.36 1 4.37 C 4.38 A 4.39 C 4.40 A
4.41 C 4.42 B 4.43 A 4.44 A 4.45 C
4.46 C 4.47 A 4.48 D 4.49 B 4.50 2
4.51 A 4.52 A 4.53 C 4.54 B 4.55 D
4.56 0 4.57 0.25 4.58 C 4.59 B 4.60 A
4.61 A 4.62 D 4.63 A 4.64 B 4.65 C
4.66 B 4.67 B 4.68 B 4.69 A 4.70 B
4.71 B 4.72 B 4.73 B 4.74 A 4.75 A
4.76 B 4.77 B 4.78 D 4.79 B 4.80 23
4.81 A 4.82 88 4.83 A 4.84 2 4.85 A
4.86 D 4.87 D 4.88 D 4.89 B 4.90 A
4.91 C 4.92 C 4.93 B 4.94 B 4.95 C
4.96 D 4.97 A 4.98 B 4.99 C 4.100 A, D
4.101 C 4.102 C 4.103 3 4.104 –6 4.105 B
4.106 B 4.107 C 4.108 1 4.109 A 4.110 A
4.111 C 4.112 D 4.113 B 4.114 A 4.115 D
4.116 B 4.117 B 4.118 D 4.119 1 4.120 A
4.121 D 4.122 A 4.123 B 4.124 C 4.125 A
4.126 A 4.127 A 4.128 B 4.129 C 4.130 A
4.131 C 4.132 A 4.133 25



 
 
 
 

5.1 A solution of the following differential (5) Non-linear first order differential
d2y dy equation
equation 2
 5  6 y  0 is given (A) A  1, B  2, C  4
dx dx
by (B) A  3, B  4, C  2
(A) y  e 2 y  e3 x (B) y  e2 x  e3 x (C) A  2, B  5, C  3
(D) A  3, B  1, C  2
(C) y  e2 x  e3 x (D) y  e2 x  e 3 x
5.4 For the differential equation
5.2 The following differential equation has
d2y
d2y
3  k2 y  0,
 dy  dx 2
3 2
 4    y2  2  x
dt  dt  the boundary conditions are
(A) degree = 2, order = 1 (i) y  0 for x  0 and
(B) degree = 1, order = 2 (ii) y  0 for x  a
(C) degree = 4, order = 3 The form of non-zero solution of y
(D) degree = 2, order = 3 (where m varies over all integers) are
5.3 Match each of the items A, B, C with an  mx 
(A) y   Am sin  
appropriate item from 1, 2, 3, 4 and 5. m  a 
d2y dy  mx 
A. a1 2  a2 y  a3 y  a4 (B) y   Am cos  
dx dx m  a 
mx
d3y (C) y   Am e a
B. a1 3  a2 y  a3
dx m
 mx
d2y dy (D) y   Am e a
C. a1 2  a2 y  a3 x 2 y  0
dx dx m

(1) Non-linear differential equation 5.5 Which of the following is a solution to


(2) Linear differential equation with the differential equation,
constant coefficient d
x(t )  3x(t )  0, x(0)  2 ?
(3) Linear homogeneous differential dt
equation (A) x(t )  3et (B) x(t )  2e3t
(4) Non-linear homogeneous differential 3
(C) x(t )   t 2 (D) x(t )  3t 2
equation 2

 
2 | Engineering Mathematics  
5.6 Match each differential equation in 5.10 With initial values y (0)  y '(0)  1, the
Group I to its family of solution curves solution of the differential equation
from Group II. d2y dy
Group I Group II 2
 4  4 y  0 at x  1 is _____.
dx dx
dy y 5.11 The solution of the differential equation
P.  1. Circle
dx x d2y dy
dy y Straight 2
 2  y  0 with y (0)  y '(0)  1
Q.  2. dt dt
dx x lines is
dy x (A) (2  t )et (B) (1  2t )e t
R.  3. Hyperbolas
dx y
(C) (2  t )e t (D) (1  2t )et
dy x
S.  5.12 The general solution of the differential
dx y dy 1  cos 2 y
equation  is
(A) P – 2, Q – 3, R – 3, S – 1 dx 1  cos 2 x
(B) P – 1, Q – 3, R – 2, S – 1 (A) tan y  cot x  c (c is a constant)
(C) P – 2, Q – 1, R – 3, S – 3 (B) tan x  cot y  c (c is a constant)
(D) P – 3, Q – 2, R – 1, S – 2 (C) tan y  cot x  c (c is a constant)
5.7 The solution of differential equation
(D) tan x  cot y  c (c is a constant)
dy
 ky, y (0)  c is 5.13 The particular solution of the initial
dx
value problem given below is
(A) x  ce ky (B) x  kecy
d2y dy
(C) y  ce kx
(D) y  ce  kx
2
 12  36 y  0
dx dx
5.8 With initial condition x(1) = 0.5, the dy
solution of the differential equation, with y (0)  3 and  36
dx x 0
dx
t  x  t is (A) (3  18 x)e 6 x (B) (3  25 x)e 6 x
dt
1 1 (C) (3  20 x)e 6 x (D) (3  12 x)e 6 x
(A) x  t  (B) x  t 2 
2 2 5.14 Consider the differential equation
t2 t d 2 x(t ) dx(t )
(C) x  (D) x  2
3  2 x(t )  0.
2 2 dt dt
5.9 If the characteristic equation of the 10
Given x(0)  20 and x(1)  , where
differential equation, e
d2y dy e  2.718, the value of x(2) is ______.
 2  y  0
dx 2
dx 5.15 Consider the differential equation
has two equal roots, then the values of dx
 10  0.2 x with initial condition
 are dt
(A)  1 (B) 0, 0 x(0)  1 . The response x(t ) for t  0 is
1 (A) 2  e0.2t (B) 2  e0.2t
(C)  j (D) 
2 (C) 50  49e0.2t (D) 50  49e0.2t

 
Differential Equation & Partial Differential Equation | 3 
5.16 Which one of the following is the 5.19 A curve passes through the point
general solution of the first order ( x  1, y  0) and satisfies the
differential equation dy x 2  y 2 y
differential equation   .
dy dx 2y x
 ( x  y  1)2
dx The equation that describes the curve is
where x, y are real ?  y2 
(A) ln 1  2   x  1
(A) y  1  x  tan 1 ( x  c) , where c is a  x 
constant. 1  y2 
(B) ln 1  2   x  1
(B) y  1  x  tan ( x  c) , where c is a 2  x 
constant.  y
(C) ln 1    x  1
(C) y  1  x  tan 1 ( x  c) , where c is a  x
constant. 1  y
(D) ln 1    x  1
(D) y  1  x  tan ( x  c) , where c is a 2  x
constant. 5.20 For the equation
x ''(t )  3x '(t )  2 x(t )  5
5.17 The general solution of the differential
the solution x(t) approaches which of the
equation,
following values as t   ?
d2y dy 5
2
 2  5y  0 (A) 0 (B)
dx dx 2
in terms of arbitrary constant K1 and (C) 5 (D) 10
K 2 is, 5.21 With K as a constant, the possible
solution for the first order differential
(A) K1e( 1 6)x
 K 2e( 1 6)x
dy 3 x
equation  e is
dx
(B) K1e( 1 8)x
 K 2e( 1 8)x
1 1
(A)  e3 x  K (B)  e3 x  K
(C) K1e(  2 6)x
 K 2 e(  2 6)x 3 3
3 x
(C) 3 e  K (D) 3 e  x  K
(D) K1e(  2 8)x
 K 2 e(  2  8)x
5.22 For the differential equation
5.18 The position of a particle y (t ) is d 2x dx
2
 6  8x  0
described by the differential equation : dt dt
with initial conditions x(0)  1 and
d2y dy 5 y
2
  dx
dt dt 4  0 , the solution is
dt t 0
The initial conditions are y(0)  1 and
dy (A) x(t )  2e 6t  e 2t
 0 . The position (accurate to two
dt t 0 (B) x(t )  2e 2t  e4t
decimal places) of the particle at t   (C) x(t )  e 6t  2e  4t
is ________. (D) x(t )  e 2t  2e 4t

 
4 | Engineering Mathematics  
5.23 A solution of the ordinary differential 5.28 Let y ( x) be the solution of the
d2y dy differential equation
equation 2
 5  6 y  0 is such
dt dt d2y dy
1  3e 2
 4  4y  0
that y (0)  2 and y (1)  . The dx dx
e3 with initial conditions y (0)  0 and
dy
value of (0) is ______. dy
dt  1 . Then the value of y (1) is
dx x 0
di
5.24 A differential equation  0.2i  0 is _______.
dt
applicable over range –10 < t < 10. If 5.29 The differential equation y '' y  0 is
i(4) = 10, then i (–5) is ______. subjected to the condition y(0)  0,
5.25 The solution for the differential equation y()  0. In order that the equation has
2
d x non-trivial solutions, the general value
 9 x with initial conditions
dt 2 of  is
dx m
x(0) = 1 and t  0  1 is (A) (B) m
dt 2
(A) t 2  t  1 m m
(C) (C)
1 2 4 6
(B) sin 3t  cos 3t 
3 3 d 2 y dy
1 5.30 The differential   sin y  0 is
(C) sin 3t  cos 3t dx 2 dx
3 (A) Linear (B) Non-linear
(D) cos 3t  t
(C) Homogeneous (D) of degree two
5.26 A function y (t ) , such that y(0)  1 and
5.31 The differential equation
y (1)  3e 1 , is a solution of the
d4y d2y
differential equation  P  ky  0 is
dx 4 dx 2
d2y dy
2
 2  y  0. (A) Linear of fourth order.
dt dt
(B) Non-Linear of fourth order.
Then y (2) is
(C) Non-homogeneous.
(A) 5e1 (B) 5e2
(D) Linear and fourth degree.
(C) 7e1 (D) 7e2 5.32 The necessary and sufficient for the
5.27 Consider the differential equation differential equation of the form
dy
(t 2  81)  5ty  sin(t ) M  x, y  dx  N  x, y  dy  0 to be exact
dt
with y (1)  2 . These exists a unique is

solution for this differential equation M N


(A) M  N (B) 
when t belongs to the interval x y
(A) (–2, 2) (B) (–10, 10) M N 2M 2 N
(C)  (D)  2
(C) (–10, 2) (D) (0, 10) y x x 2 y

 
Differential Equation & Partial Differential Equation | 5 
d2y dy 5.39 The particular integer of equation
5.33 Solve for y if  2  y  0 with
dt 2
dt d 4v
 4 4 v  1  x  x 2 is
y  0   1 and y '  0   2 . dx 4

(A) 2et  tet (B) et  2te2t 1  x  x3 1  x  x2


(A) (B)
4 4 4 4
(C) et  3te2t (D) et  3tet
dy 1  x2  x4 1  x  x2
5.34 For the differential equation  5y  0 (C) (D)
dt 4 4
with y(0)  1 , the general solution is 5.40 The general solution of the differential
d2y dy
(A) e5t (B) e5t equation x 2 2
 x  y  0 is
dx dx
(C) 5e5t (D) e 5t
(A) Ax  Bx 2 (A, B are constants)
5.35 The differential equation
(B) Ax  B log x (A, B are constants)
y ''  x3 sin x 5  y ' y  cos x 3 is
(C) Ax  Bx 2 log x (A, B are constants)
(A) Homogeneous
(D) Ax  Bx log x (A, B are constants)
(B) Non-linear
(C) 2nd order linear 5.41 The solution of differential equation
y " 3 y ' 2 y  0 is of the form
(D) Non-homogeneous with constant
coefficients (A) C1e x  C2 e 2 x
5.36 The solution of differential equation (B) C1e  x  C2 e3 x
f "( x)  4 f '( x)  4 f ( x)  0 .
2 x
(C) C1e  x  C2 e 2 x
(A) f1 ( x)  e
2 x
(D) C1e 2 x  C2 2 x
(B) f1 ( x )  e , f 2 ( x )  e
2x

dy 2 ln( x)
(C) f1 ( x )  e 2 x , f 2 ( x )  xe 2 x 5.42 If x 2  2 xy  , and y (1)  0,
dx x
(D) f1 ( x )  e 2 x , f 2 ( x )  e  x then what is y(e)?
5.37 The one dimensional heat condition (A) e (B) 1
T  2T 1 1
partial differential equation  is (C) (D) 2
t x 2 e e
(A) Parabola (B) Hyperbolic d2y dy
5.43 For 2
 4  3 y  3e 2 x , the
(C) Elliptic (D) Mixed dx dx
5.38 The particular solution for the particular integral is
differential equation 1 2x 1
(A) e (B) e 2 x
d2y dy 15 5
2
 3  2 y  5cos x is
dt dx (C) 3e2 x (D) c1e  x  c2 e 3 x
(A) 0.5cos x  1.5sin x 5.44 The partial differential equation
(B) 1.5cos x  0.5sin x          
2 2
        0 has
(C) 1.5sin x x 2 y 2  x   y 

 
6 | Engineering Mathematics  
(A) degree 1, order 2 d  x   4  5  x 
(C) 
(B) degree 1, order 1 dt  y   3 8   y 
(C) degree 2, order 1
d  x  4 8   x 
(D) degree 2, order 2 (D) 
dt  y   3  5  y 

5.45 Given that x  3 x  0, x (0)  1, x (0)  0,
5.50 Consider two solutions x(t )  x1 (t ) and
what is x(1) ?
x(t )  x2 (t ) of the differential equation
(A) – 0.99 (B) – 0.16
(C) 0.16 (D) 0.99 d 2 x(t )
 x(t )  0, t  0, such that
dy dt 2
5.46 The solution of x  y  x 4 with the dx1 (t )
dx x1 (0)  1,  0, x2 (0)  0,
6 dt t 0
condition y (1)  is
5 dx2 (t )
 1.
x4 1 4 x4 4 dt t 0
(A) y   (B) y  
5 x 5 5x The Wronskian
4 5
x x x1 (t ) x2 (t )
(C) y  1 (D) y   1
5 5 W (t )  dx1 (t ) dx2 (t ) at t   / 2 is
5.47 The partial differential equation dt dt
u u  2u (A) 1 (B) –1
u  is a
dt x x 2

(A) Linear equation of order 2 (C) 0 (D)
2
(B) Non-linear equation of order 1
5.51 Consider the following partial
(C) Linear equation of order 1
differential equations for u ( x, y ) with
(D) Non-linear equation of order 2
the constant c  1 :
d2y
5.48 If y  f ( x) is the solution of 0 u u
dx 2 c 0
with the boundary conditions y  5 at y x
dy Solution of this equation is
x  0, and  2 at x  10, f (15) 
dx (A) u ( x, y )  f ( x  cy )
________.
(B) u ( x, y )  f ( x  cy )
5.49 The matrix form of the linear system
dx dy (C) u ( x, y )  f (cx  y )
 3x  5 y and  4 x  8 y is,
dt dt (D) u ( x, y )  f (cx  y )
d  x   3  5  x  5.52 If y  f ( x) satisfies the boundary value
(A) 
dt  y   4 8   y  problem y " 9 y  0, y (0)  0,
d  x  3 8   x   
(B) 
dt  y   4  5  y  y    2 , then y   is ______.
2 4

 
Differential Equation & Partial Differential Equation | 7 
5.53 Consider the following differential 5.58 Biotransformation of an organic
dy compound having concentration (x) can
equation   5 y ; initial condition :
dt be modeled using an ordinary
y  2 at t  0 . The value of y at t  3 is dx
differential equation  kx 2  0, where
dt
k is the reaction rate constant. If x = a at
(A)  5e10 (B) 2e 10
t = 0, the solution of the equation is
(C) 2e 15 (D) 15e2 1 1
(A) x  ae kt (B)   kt
5.54 Given the ordinary differential equation x a
d 2 y dy (C) x  a (1  e  kt ) (D) x  a  kt
  6 y  0 with y (0)  0 and
dx 2 dx 5.59 Transformation of linear form by
dy
(0)  1 , the value of y (1) is ______ substituting v  y1 n of the equation
dx
dy
(correct to two decimal places).  p(t ) y  q(t ) y n ; n  0 will be
dt
5.55 Consider the differential equation
3 y "( x)  27 y ( x)  0 with initial dv
(A)  (1  n) pv  (1  n)q
dt
conditions y (0)  0 and y '(0)  2000 .
dv
The value of y at x  1 is _______. (B)  (1  n) pv  (1  n)q
dt
5.56 The solution for the following
differential equation with boundary dv
(C)  (1  n) pv  (1  n)q
conditions y (0)  2 and y '(1)   3 is, dt
d2y dv
where  3x  2 . (D)  (1  n) pv  (1  n)q
dx 2 dt

x3 x 2 d2y dy
(A) y    3 x  6 5.60 The solution of 2
 2  17 y  0 ;
3 2 dx dx
 dy 
x2 y(0)  1 ,    0 in the range
(B) y  3x   5 x  2
3
 dx  x  
2 4

x3 5x 
(C) y   x2   2 0 x is given by
2 2 4

x2 3  1 
(D) y  x 3   5x  (A) e  x  cos 4 x  sin 4 x 
2 2  4 
5.57 The number of boundary conditions  1 
(B) e x  cos 4 x  sin 4 x 
required to solve the differential  4 
 2  2  1 
equation   0 is (C) e 4 x  cos 4 x  sin x 
x 2 y 2  4 
(A) 2 (B) 0  1 
(D) e 4 x  cos 4 x  sin 4 x 
(C) 4 (D) 1  4 

 
8 | Engineering Mathematics  
5.61 The solution of the differential equation 5.65 The type of partial differential equation
dy 2 P 2 P 2 P P P
x 2  2 xy  x  1  0 , given that at   3 2   0 is
dx x 2
y 2
xy x y
x  1 , y  0 is (A) Elliptic (B) Parabolic
1 1 1 (C) Hyperbolic (D) None of these
(A)   5.66 The respective expressions for
2 x 2 x2
1 1 1 complementary function and particular
(B)   2 integral part of the solution of the
2 x 2x
differential equation
5.62 The solution for the differential equation
d4y d2y
dy  3  108 x 2 are
 x 2 y with condition that y = 1 at x dx 4 dx 2
dx
= 0 is (A) [c1  c2 x  c3 sin 3 x  c4 cos 3 x]
1 and [3x 4  12 x 2  c]
(A) y  e 2x
(B) [c2 x  c3 sin 3 x  c4 cos 3 x ] and
x3
(B) ln( y )   4 [5 x 4  12 x 2  c]
3
(C) [c1  c3 sin 3 x  c4 cos 3 x] and
x2
(C) ln( y )  [3x 4  12 x 2  c]
2
x3 (D) [c1  c2 x  c3 sin 3 x  c4 cos 3 x]
(D) y  e 3
and [5 x 4  12 x 2  c]
dy x 5.67 Consider the following second order
5.63 Solution of  at x  1 and
dx y linear differential equation
y  3 is d2y
 12 x 2  24 x  20
dx 2
(A) x  y 2  2 (B) x  y 2  4
The boundary conditions are :
(C) x 2  y 2   2 (D) x 2  y 2  4 x  0, y  5 and at x  2, y  21 .
5.64 The solution of the partial differential The value of y at x  1 is______.
u u 2
5.68 Consider the following difference
equation   2 is of the form
t x equation
y y
(A) C cos(kt ) C1e( k / ) x
 C2 e  ( k / ) x  x( y dx  x dy ) cos  y ( x dy  y dx)sin
  x x
Which of the following is the solution of
(B) Cekt C1e( k / ) x
 C2 e  ( k / ) x 
  the above equation (c is an arbitrary
constant)?
(C) Ce kt C1 cos( k /  ) x
x y x y
(A) cos  C (B) sin  C
 C2 sin( k /  ) x  y x y x
y y
(D) C sin(kt ) C1 cos( k /  ) x (C) xy cos C (D) xy sin C
x x

 
Differential Equation & Partial Differential Equation | 9 
5.69 Consider the following partial places) of flow at a downstream section
differential equation at x  1m from one calculation step of
 2  2  2 Single Step Euler Method is ______.
3  B  3  4  0 5.74 The differential equation
x 2 xy y 2
2
For this equation to be classified as  dy  d2y
   y 0
parabolic, the value of B 2 must be  dx  dx 2
_____. can be reduced to (where  is a
dQ constant).
5.70 The solution of the equation Q 1
dt  dy 
3
3y 2

with Q  0 at t  0 is (A)     
 dx  2
(A) Q(t )  e t  1 (B) Q(t )  1  e t  dy 
2

(B)      2 y
(C) Q(t )  1  et (D) Q(t )  1  e t  dx 
5.71 The value of y as t   for the dy 
(C) 
following differential equation for an dx y 2
initial value of y(1)  0 is dy 
(D) 
dy dx y
(4t 2  1)  8 yt  t  0
dt 5.75 Match the following, where x is the
1 spatial coordinate and t is time
(A) 1 (B)
2 Group - I Group - II
1 1 c c
(C) (D) P. Wave equation 1. 
4 8 t x
5.72 The solution (up to three decimal c 2  c
2
Q. Heat equation 2. 
places) at x  1 of the differential t x 2
d2y dy  2c c
equation  2  y  0 subject to 3.  2
dx 2
dx t 2
x
boundary conditions y(0)  1 and  2c 2  c
2
4.  
dy t 2 x 2
(0)  1 is _____.
dx (A) P - 4, Q - 2 (B) P - 2, Q - 4
5.73 Variation of water depth (y) in a (C) P -3, Q-1 (D) P -1, Q -3
gradually varied open channel flow is
5.76 The differential equation,
given by the first order differential
equation d2y dy
2
 sin x  ye x  sinh x is
10 dx dx
 ln( y )
dy 1 e 3 (A) first order and linear

dx 250  45e 3ln( y ) (B) first order and non-linear
Given initial conditions : y ( x  0) = 0.8 (C) second order and linear
m. The depth (in m, up to three decimal (D) second order and non-linear

 
10 | Engineering Mathematics  
5.77 If f ( x) is the solution of the equation (B) Linear differential equation of first
degree
dy
 2 xy  2 x  0 (C) Linear differential equation of
dx
second degree
and g ( x) is the solution of the equation
(D) Non-linear differential equation of
dy second degree
 2 xy  2 x  0
dx
and the constant of integration in f ( x)
is equal to that in g ( x) , then which of
the following is true?
(A) g ( x)  f ( x)  2
(B) g ( x)  f ( x)  1
(C) g ( x)  f ( x)  1
(D) g ( x)  f ( x)  2
Common Data for Question 5.78 and 5.79
5.78 The complete solution of the ordinary
differential equation
d2y dy
2
 p q y 0
dx dx
is y  C1e  x  C2 e 3 x then p and q are
(A) p  3, q  3 (B) p  3, q  4
(C) p  4, q  3 (D) p  4, q  4
5.79 Which of the following is a solution of
the differential equation
d2y dy
2
 p   q  1 y  0?
dx dx
(A) e3x (B) xe x
(C) xe2 x (D) x 2 e2 x
5.80 The differential equation
d 2 y dy
2
  x3 y  e x is a
dx dx
(A) Non-linear differential equation of
first degree

 
Differential Equation & Partial Differential Equation | 11 

Answer Key : Differential Equation & Partial Differential Equation


5.1 B 5.2 B 5.3 A 5.4 A 5.5 B
5.6 A 5.7 C 5.8 D 5.9 A 5.10 0.541
5.11 B 5.12 C 5.13 A 5.14 0.86 5.15 C
5.16 D 5.17 A 5.18 – 0.21 5.19 A 5.20 B
5.21 A 5.22 B 5.23 –3 5.24 1.652 5.25 C
5.26 B 5.27 A 5.28 7.38 5.29 B 5.30 B
5.31 A 5.32 C 5.33 D 5.34 B 5.35 C
5.36 C 5.37 A 5.38 A 5.39 B 5.40 D
5.41 C 5.42 D 5.43 B 5.44 A 5.45 B
5.46 A 5.47 D 5.48 35 5.49 A 5.50 A
5.51 B 5.52 –1 5.53 C 5.54 1.47 5.55 94.08
5.56 C 5.57 C 5.58 B 5.59 A 5.60 A
5.61 A 5.62 D 5.63 D 5.64 B 5.65 C
5.66 A 5.67 18 5.68 C 5.69 36 5.70 D
5.71 D 5.72 0.368 5.73 0.793 5.74 D 5.75 A
5.76 D 5.77 A 5.78 C 5.79 C 5.80 A



 
6.1 The Laplace transform of the periodic m s
(C) (D)
function f (t ) described by the curve s  m2
2
s  m2
2

below i.e. 6.3 Solve the initial value problem


sin t , if (2n  1)  t  2n d2y dy
  4  3 y  0 with y  3 and
f (t )   (n  1, 2,3,...) is dx 2
dx
0, otherwise
 dy
 7 at x  0 using Laplace
f (t ) dx
transform technique?
p 2p 3p 4p 5p 6p 7p 8p
t (A) e x  2e 2 x (B) e x  2e3 x
0
(C) 2e x  e3 x (D) e x  3e 2 x
e s 6.4 ( s  1)2 is Laplace transform of
(A) 2
( s  1) (e s  1) (A) t 2 (B) t 3
e s (C) e2t (D) tet
(B) 2
( s  1) (e s  1)
6.5 The Laplace transform of the function
e s sin 2 2t is
(C) 2
( s  1) (e s  1)
1 s  s
s (A)    (B) 2
e  2 s 2( s  16) 
2
s  16
(D)
( s  1) (e s  1)
2

1 s s
6.2 If f (t ) is a finite and continuous (C)  2 (D)
s ( s  4) s 4
2

function for t  0 the Laplace transform


6.6 Using Laplace transform, solve
is given by
  d2y 
F  e  st
f (t ) dt ,  2   4 y  12t , given y0 and
0
 dt 
then for f (t )  cosh mt , the Laplace dy
 9 at t  0
transform is dt
s m (A) 3  3sin 2t (B) 3t  3sin 2t
(A) 2 (B)
s  m2 s  m2
2
(C) 3  3cos 2t (D) 3t  3cos 2t
2 | Engineering Mathematics
6.7 Laplace transform of the function sin t 6.12 Laplace transform of cos (t ) is
is s
. The Laplace transform of
s  s  2
2
(A) 2 (B) 2
s  2
s  2 e2t cos (4t ) is
s 
(C) 2 (D) 2 s2 s2
s  2
s  2 (A) (B)
6.8 A delay unit step function is defined as ( s  2)2  16 ( s  2)2  16
0 for t  a s2 s2
u (t  a )   (C) (D)
1 for t  a ( s  2)2  16 ( s  2)2  16
Its Laplace transform is 6.13 The Laplace transform of e i 5t where
 as
e
(A) ae  as (B) i  1, is
s
e as e as s  5i s  5i
(C) (D) (A) (B)
s a s 2  25 s 2  25
6.9 If F ( s ) is the Laplace transform of s  5i s  5i
(C) (D)
function f (t ), the Laplace transform of s 2  25 s 2  25
t 6.14 Laplace transform of the function f (t )
0
f () d  is

1 1 is given by L{ f (t )}   f (t ) e  st dt.
(A) F ( s) (B) F ( s )  f (0) 0
s s
Laplace transform of the function shown
(C) sF ( s)  f (0) (D)  F ( s ) ds
below is given by,
6.10 The inverse Laplace transform of
f (t )
1
is 2
(s  s)
2

(A) 1  et (B) 1  et
(C) 1  e  t (D) 1  e  t t
0 1
6.11 The function f (t ) satisfies the
1  e 2 s 1  e s
d f 2 (A) (B)
differential equation  f  0 and s 2s
dt 2
the auxiliary conditions, 2  2e  s 1  2e  s
(C) (D)
df s s
f (0)  0, (0)  4 .
dt 6.15 If f (t ) is a function defined for all
The Laplace transform of f (t ) is given t  0, its Laplace transform F ( s ) is
by defined as
2 4  
(A) (B) (A)  e st f (t ) dt
s 1 s 1 0
(B)  0
e  st f (t ) dt
4 4  
(C) 2
s 1
(D) 4
s 1
(C) 0
eist f (t ) dt (D)  e  ist f (t ) dt
0
Laplace Transform | 3
6.16 The Laplace transform of tet is 6.22 The inverse Laplace transform of
s 1 1
(A) (B) is
( s  1)2 ( s  1) 2 (s  2s)
2

1 s 2 t (1  e 2t )
(C) (D) (A) (1  e ) (B)
( s  1)2 s 1 2
(1  e 2 t ) (1  e 2 t )
6.17 F ( s ) is the Laplace transform of the (C) (D)
2 2
function f (t )  2t 2 et . 6.23 Laplace transform of f ( x )  cosh ( ax)
F (1) is _______ (correct to two decimal is
places). a s
(A) (B) 2
6.18 The Laplace transform of the function s a 2
2
s  a2
f (t )  et when t  0 and where a is a a s
(C) 2 (D) 2
constant is s a 2
s  a2
1 1 6.24 Find the Laplace transform of the
(A) (B)
(s  a) (s  a) following input function shown in figure
1 1 1
(C) (D)
( s  a ) 1 ( s  a ) 1 1 2 3 4 5
0
6.19 The inverse Laplace transform of
s9 –1
is
s  6 s  13
2
(1  e  s ) 2 (1  e  s )
(A) cos 2t  9sin 2t (A) (B)
s (1  e 2 s ) s
(B) e 3t cos 2t  3e 3t sin 2t
(1  e s ) (1  e  s )e  s
(C) e 3 t
cos 2t  3e 3 t
cos 2t (C) (D)
s (1  e  s ) s (1  e 2 s )
3 t 3 t
(D) e cos 2t  e sin 2t 6.25 For the time domain f (t )  t the
6.20 Laplace transform L( f '), where f ' is t

the derivative of function f is given by Laplace transform of  f (t ) dt


0
is given
(A) sL( f )  f (0)
by
(B) L( f )  f (0)
1 2
(A) (B) 3
(C) s L( f )  f (0)
2
2s 3
s
L( f ) 1 2
(D)  f (0) (C) 3 (D) 2
s s s
6.21 The Laplace transform of a unit step 6.26 Given that the Laplace transform of the
function ua (t ), defined as function below over a single period
1
e  as 0  t  2 is 2 (1  e  s ) 2, the Laplace
(A) (B) se  as s
s transform of the periodic function over
(C) s  u (0) (D) se  as  1 0  t   is
4 | Engineering Mathematics
f (t )

t
0 2 4
1 1
(A) (1  e  s ) 2 (B) (1  e  s ) 2
s s
1(1  e  s ) 1 s
(C) (D) tanh
s 2 (1  e  s ) s 2
6.27 The inverse Laplace transform of
1
is
2s  3s  1
2

(A) e  t /2  e  t (B) 2e  t /2  e  t
(C) e  t  2e  t /2 (D) e  t  e  t /2
6.28 The Laplace transform of the function
shown in figure below is
f (t )

t
a b

Ve ( a  b ) s V bs  as
(A) (B) (e  e )
s s
V V
(C) (e  as  e  bs ) (D) (e as  ebs )
s s
6.29 For the time domain function, f (t )  t 2
which ONE of the following is the
Laplace transform?
3 1
(A) 4 (B) 2
s 4s
2 2
(C) 2 (D) 3
s s
6.30 The Laplace transform of eat sin (bt ) is
b (s  a)
(A) (B)
(s  a)2  b2 (s  a)2  b2
(s  a) b
(C) (D)
(s  a)2  b2 (s  a)2  b2
Laplace Transform | 5

Answer Key : Laplace Transform


6.1 D 6.2 D 6.3 B 6.4 D 6.5 A
6.6 B 6.7 B 6.8 B 6.9 A 6.10 C
6.11 C 6.12 D 6.13 B 6.14 C 6.15 B
6.16 B 6.17 0.5 6.18 A 6.19 B 6.20 A
6.21 A 6.22 D 6.23 B 6.24 C 6.25 C
6.26 C 6.27 A 6.28 C 6.29 D 6.30 A


 
 
 

7.1 Consider the analytic functions 7.5 If f ( x  iy)  x3  3 xy 2  i ( x, y), where


f ( z )  x 2  y 2  i 2 xy i  1 and f ( x  iy) is an analytic
of the complex variable z  x  iy, where function, then  ( x, y ) is

i  1. The derivative f '( z ) is (A) y3  3 x2 y (B) 3 x2 y  y3

(A) 2 x  i 2 y (B) x 2  iy 2 (C) x4  4 x3 y (D) xy  y 2

(C) x  iy (D) 2 x  i 2 y 7.6 If a complex number  satisfies the


equation 3  1, then the value of
7.2 The product of a complex number
z  x  iy and its complex conjugate z 1
1   is

is
(A) 0 (B) 1
(A) x2 (B) y 2 (C) 2 (D) 4
(C) x2  y 2 (D) x 2  y 2 7.7 The product of the complex numbers
(3  i 2) and (3  i 4) results in
z2 1
7.3 The function f ( z )  is singular (A) 1  i 6 (B) 9  i8
z2  4
at (C) 9  i8 (D) 17  i 6
(A) z   2 (B) z  1  5  i10
7.8 The value of the expression is
3  i4
(C) z   i (D) z   2i
(A) 1  i 2 (B) 1  i 2
2
z (C) 2  i (D) 2  i
7.4 The value of  z 4
1
dz using Cauchy’s
C 3 1
7.9 If a complex number z   i then
integral around the circle z  1  1, 2 2
where z  x  iy, is z 4 is

 i 1 3
(A) 2 i (B) (A) 2 2  2i (B) i
2 2 2
 3 i 3 1 3 1
(C) (D)  2i (C) i (D) i
2 2 2 8 8

 
2 | Engineering Mathematics  
7.10 The function 1
7.14 For the function f ( z )  .
1  y (2  z ) ( z  2)
w  u  iv  log ( x 2  y 2 )  i tan 1  
2  x The residue at z  2 is _______.
is not analytic at the point. 7.15 If i  1, the value of the integral
7z  i
(A) 0, 0 (B) 0, 1
C z ( z 2  1) dz z 2
(C) 1, 0 (D) 2, 
Using the Cauchy-Residue theorem is
7.11 A complex-valued f ( z ),
function,
(A) 2 i (B) 0
given below is analytic in domain D :
(C)  6  (D) 6 
f ( z )  u ( x, y)  iv ( x, y)
7.16 Let j  1 . Then the value of j j is
z  x  iy
(A) j (B) –1
Which of the following is NOT correct?
(C)  /2 (D) e /2
df v u 7.17 Using the residue theorem, the value of
(A)  i
dz y y the integral (counter clockwise)
df u v 8  7z
(B)  i  z  4 dz around a circle with center
dz x x
at z  0 and radius  8 (where z is a
df v u
(C)  i complex number and i  1 ), is
dz y y
(A)  20 i (B)  40 
df v u (C)  40 i (D) 40 i
(D)  i
dz y x
7.18 An analytic function w ( z ) is defined as
7.12 What are the modulus (r ) and () of
w  u  iv, where i  1 and z  x  iy.
the complex number 3  4i ? y
If the real part is given by u  ,
4 x  y2
2
(A) r  7,   tan 1  
3 w ( z ) is
3 1 1
(B) r  7,   tan 1   (A) (B) 2
4 z z
i 1
3 (C) (D)
(C) r  5,   tan 1   z iz
4
7.19 The value of the contour integral
4 1 ez
(D) r  5,   tan 1  
3
I  ( z  1) ( z  3) dz
2i
C
7.13 For complex variable z, the value of the where, C is the circle z  2 is
2 z
1 e
contour integral 
2i C z ( z  3)
dz along
(A)
1
(B)
11 1 
  
2e 2  e e3 
the clockwise contour C : z  2 (up to 1
(C) Zero (D)
two decimal places) is _______. (2 i) e3

 
Complex Variables | 3 
7.20 If z  x  iy is a complex number, 7.26 If z  x  jy, where x and y are real, the

where i  1 then which of the value of e jz is


following is an analytic function of z?
x2  y 2
(A) 1 (B) e
(A) x  y
2 2
(B) 2ixy
(C) e y (D) e  y
(C) x  y  2 ixy
2 2
(D) x  y  2 ixy
2 2

sin z
1
7.27 The value of  z
dz where the
7.21 The value of  z 2
dz , where the contour
C contour of integration is a simple closed
is the unit circle traversed clockwise, is curve around the origin is,
(A)  2 i (B) 0 (A) 0 (B) 2 j
(C) 2 i (D) 4 i 1
(C)  (D)
1 z2 1 2 j
2j C z 2  1
7.22 The value of the integral dz
7.28 One of the roots of the equation x3  j,
where z is a complex number and C is a
where j is the positive square root of –1
unit circle with center at 1  0 j in the
is
complex plane is _______.
3 1
(A) j (B) j
7.23 Let z  x  jy where j  1. Then 2 2
cos z  3 1  3 1
(C) j (D) j
(A) cos z (B) cos z 2 2 2 2
(C) sin z (D) sin z 7.29 The contour C in the adjoining figure is
7.24 The value of (1  i )8 where i  1 described by x2  y 2  16. The value of
(A) 8  4i (B) 8  4 j z2  8
(C) 16 (D) 8
C 0.5 z  1.5 j dz is
1 2
7.25 Given f ( z )   . If C is a
z 1 z  3
counterclockwise path in the z-plane
such that z  1  1, the value of

1
 f ( z ) dz is
2 j
C
Note : j  1

(A) – 2 (B) – 1 (A)  2 j (B) 2 j


(C) 1 (D) 2 (C) 4 j (D)  4 j

 
4 | Engineering Mathematics  
7.30 A complex variable z  x  j (0.1) has 7.32 The value of the integral of the complex
its real part x varying in the range   3s  4
function f ( s )  . Along the
( s  1) ( s  2)
to  . Which one of the following is the
1 path s  3 is
locus (shown in thick lines) of in the
z
(A) 2 j (B) 4 j
complex plane?
(C) 6 j (D) 8 j
(A)
7.33 Consider the circle z  5  5i  2 in the
complex number plane ( x, y ) with
z  x  iy . The minimum distance from
the origin to the circle is
(B) (A) 5 2  2 (B) 54
(C) 34 (D) 5 2
7.34 Let z 3  z , where z is a complex not
equal to zero. Then z is a solution of
(A) z 2  1 (B) z 3  1
(C)
(B) z 4  1 (C) z 9  1
7.35 Consider the following complex
function,
9
f ( z) 
( z  1) ( z  2) 2
(D)
Which of the following is one of the
residues of the above function?
9
(A) –1 (B)
16
(B) 2 (D) 9
sin z 7.36 For an analytic function,
7.31 For the function of a complex
z3
f ( x, iy )  u ( x, y)  iv ( x, y )
variable z, the point z  0 is
u is given u  3x 2  3 y 2 . The
by
(A) A pole of order 3 expression for v, considering K to be a
(B) A pole of order 2 constant is
(C) A pole of order 1 (A) 3 y 2  3x 2  K (B) 6 x  6 y  K
(D) Not a singularity (C) 6 y  6 x  K (D) 6xy  K

 
Complex Variables | 5 
7.37 The value of the integral (A) I  0; Singularities set  
cos (2 z ) (B) I  0; Singularities set
 (2 z  1) ( z  3) dz
C
 (2n  1)  
(where C is a closed curve given by   , n  0,1, 2....
 2 
z  1 ) is

(C) I  ; Singularities set
i 2
(A)  i (B)
5
  n; n  0,1, 2....
2 i
(C) (D) i (D) None of the above
5
7.38 Potential function  is given   x 2  y 2 . 7.41 ex is a periodic with a period of
(A) 2  (B) 2 i
What will be the stream function 
(C)  (D) i
with the condition   0 at x  0, y  0?
7.42 F ( z ) is a function of the complex
(A) 2 xy (B) x  y
2 2
variable z  x  iy given by
(C) x 2  y 2 (D) 2 x 2 y 2 F ( z )  iz  k Re ( z )  i Im ( z )

7.39 Using Cauchy’s integral theorem, the For what value of k will F ( z ) satisfy
the Cauchy-Riemann equation?
value of the integral (integration being
(A) 0 (B) 1
taken in counter clockwise direction)
(C) –1 (D) y
z3  6 7.43 Let z be a complex variable. For a
C 3z  i dz is where C is z  1 counter-clockwise integration around a
unit circle C centered at origin.
2 
(A)  4 i (B)  6 i 1
81 8  5 z  4 dz  Ai
C

4
(C)  6 i (D) 1 Then the value of A is
81
2 4
7.40 Consider likely applicability of (A) (B)
5 2
Cauchy’s integral theorem to evaluate 4
(C) 2 (D)
the following integral counter clockwise 5
around the unit circle C, 7.44 If f ( z )  ( x 2  ay 2 )  i bxy is a complex
analytic function of z  x  iy, where
 sec z dz
I 
C i  1, then
z being a complex variable. The value of (A) a  1, b  1 (B) a  1, b  2
I will be (C) a  1, b  2 (D) a  2, b  2

 
6 | Engineering Mathematics  
3z  5 7.48 Given two complex numbers
7.45 The value of  ( z  1) ( z  2) dz

along a
2
z1  5  (5 3) i and z2   2 i,
closed path  is equal to (4 i ), where 3
z  x  iy and i  1 . The correct path the argument of
z1
in degree is
 is z2
(A) (A) 0 (B) 30
(C) 60 (D) 90
7.49 If z is a complex variable, the vector of
3i
dz
(B) 
5
z
is

(A)  0.511  1.57 i


(B)  0.511  1.57 i
(C) (C) 0.511  1.57 i
(D) 0.511  1.57 i
7.50 An analytic function of a complex
variable z  x  iy is expressed as
(D)
f ( z )  u ( x, y )  iv ( x, y )
where i  1 . If u ( x, y )  x 2 y 2 , then
expressed for v ( x, y ) in terms of x , y
7.46 A function f of the complex variable and a general constant c would be
z  x  iy, is given as x2  y 2
(A) xy  c (B) c
f ( x, y )  u ( x, y )  iv ( x, y ), 2
where u ( x, y )  2 kxy , v ( x, y )  x 2  y 2 . ( x  y)2
(C) 2xy  c (D) c
The value of k, for which the function is 2
analytic, is _______. 7.51 An analytic function of a complex
7.47 f ( z )  u ( x, y )  iv ( x, y ) is an analytic variable z  x  iy is expressed as
function of complex variable z  x  iy f ( z )  u ( x, y )  iv ( x, y )
where i  1 . If u ( x, y)  2 xy, then where i  1 if u ( x, y)  2 xy, then
v ( x, y ) may be expressed as v ( x, y ) must be
(A)  x 2  y 2  constant (A) x 2  y 2  constant
(B) x 2  y 2  constant (B) x 2  y 2  constant
(C) x 2  y 2  constant (C)  x 2  y 2  constant
(D)  ( x 2  y 2 )  constant (D)  x 2  y 2  constant

 
Complex Variables | 7 
7.52 The modulus of the complex number 7.57 The value of the contour integral in the
 3  4i  complex plane
  is
 1  2i 
z3  2z  3
(A) 5 (B) 5  z  2 dz
1 1
(C) (D) along the contour z  3, taken counter
5 5
7.53 An analytic function of a complex clockwise is
variable z  x  iy is expressed as (A) 18 i (B) 0
f ( z )  u ( x, y )  iv ( x, y ) where i  1 . (C) 14 i (D) 48 i
If u  xy , the expression for v should be
7.58 Consider the line integral
( x  y)2 x2  y 2
(A) k (B) k I   ( x 2  i y 2 ) dz where z  x  iy
2 2 C
y 2  x2 ( x  y)2
(C) k (D) k The line C is shown in the figure below.
2 2
7.54 Assuming i  1 and t is real number,
 /3

 e dt
it
is
0

3 1 3 1
(A) i (B) i
2 2 2 2
The value of I is
1  3 1  3
(C)  i   (D)  i  1  
2  2  2  2  1 2
(A) i (B) i
2 3
7.55 If C is a circle z  4 and
3 4
z2 (C) i (D) i
f ( z)  2
( z  3 z  2)
, then  f ( z) dz
C
is 4 5
7.59 The value of the integral
(A) 1 (B) 0
2z  5
(C) – 1 (D) – 2
  1 2
dz
z 1 C
 z   ( z  4 z  5)
7.56 The value of the integral C z 2  4 dz in  2

counter clockwise direction around a Over the contour z  1, taken in the


circle C of radius 1 with center at the
anti-clockwise direction, would be
point z   2 is
i 24 i 48 i
(A) (B) 2 i (A) (B)
2 13 13
i 24 12
(C)  (D)  2 i (C) (D)
2 13 13

 
8 | Engineering Mathematics  
7.60 Given f ( z )  g ( z )  h( z ), where f , g , h 7.63 Integration of the complex function
are complex valued functions of a z2
f ( z)  , in the counter clockwise
complex variable z. Which one of the z 2 1
following statements is TRUE?
direction around z  1  1 is
(A) If f ( z ) is differentiable at z0 , then
(A)  i (B) 0
g ( z ) and h( z ) are also differentiable
at z0 . (C) i (D) 2i

(B) If g ( z ) and h( z ) are differentiable 7.64 Square roots of  i, where i  1, are
at z0 , then f ( z ) is also differentiable (A) i,  1
at z0 .    
(B) cos     i sin    ,
(C) If f ( z ) is continuous at z0 , then it  4  4

is differentiable at z0 .  3   3 
cos    i sin  
(D) If f ( z ) is differentiable at z0 , then  4   4 

so are its real and imaginary parts.   3 


(C) cos    i sin   ,
7.61 All the values of the multi-valued 4  4 

complex function 1i , where i  1, are  3  


cos    i sin  
(A) purely imaginary.  4  4
(B) real and non-negative.  3   3 
(D) cos    i sin    ,
(C) on the unit circle.  4   4 
(D) equal in real and imaginary parts.  3   3 
cos     i sin  
7.62 Let S be the set of points in the complex  4   4 
plane corresponding to the unit circle.
7.65 A point z has been plotted in the complex
(That is S  {z : z  1} ). Consider the
plane, as shown in figure below,
function f ( z )  zz*, where z * denotes
the complex conjugate of z. The f ( z )
maps S to which one of the following in
the complex plane
(A) Unit circle.
(B) Horizontal axis line segment from
origin to (1, 0).
1
(C) The point (1, 0). The plot of the complex number y 
z
(D) The entire horizontal axis.
is
 
Complex Variables | 9 
(A) The value of the integral
1  1 
   dz is _______.
j C  z 2  1 
dz
7.68 The value of C (1  z 2 ) where C is the
i
contour z   1 is
(B) 2
(A) 2i (B) 
1
(C) tan z (D) i tan 1 z
7.69 The residues of a function
1
f ( z)  are
( z  4) ( z  1)3
1 1
(C) (A)  and 
27 125
1 1
(B) and 
125 125
1 1
(C)  and
27 5
1 1
(D) and 
(D) 125 5
7.70 An integral I over a counterclockwise
circle C is given by,
z2 1 z
I  z 2  1 e dz
C

If C is defined as z  3, then the value


z of I is
7.66 Given X ( z )  with z  a, the (A)  i sin (1) (B)  2i sin (1)
( z  a)2
(C)  3i sin (1) (D)  4i sin (1)
residue of X ( z ) z n 1 at z  a for n  0
1 ez
will be 7.71 The values of the integral  z  2 dz
2i
(A) a n 1 (B) a n C

along a closed contour C in anti-


(C) n a n (D) n a n 1
clockwise direction for
7.67 The contour C given below is one the (i) The point z0  2 inside the contour
complex plane z  x  jy , where j   1. C and
(ii) The point z0  2 outside the contour
C
respectively, are
(A) (i) 2.72 (ii) 0 (B) (i) 7.39 (ii) 0
(C) (i) 0 (ii) 2.72 (D) (i) 0 (ii) 7.39

 
10 | Engineering Mathematics  
sin ( z ) 7.79 The residues of a complex function
7.72 For f ( z )  , the residue of the
z2 1 2z
X ( z)  at its poles are
pole at z  0 is _______. z ( z  1) ( z  2)
7.73 Consider the complex valued function
3 1 1 1 1
f ( z )  2 z 3  b z where z is a complex (A) , and 1 (B) , and –1
2 2 2 2
variable. The value of b for which the
1 3 1 3
function f ( z ) is analytic is _______. (C) ,1 and (D) , 1 and
2 2 2 2
7.74 In the following integral, the contour C
encloses the points 2i and  2i 7.80 If f ( z )  c0  c1 z 1 , then
1 sin z 1  f ( z)
 

2 C ( z  2i )3
dz 
unit
z
dz is given by
circle
The value of the integral is _______.
7.75 If C denotes the counterclockwise unit (A) 2 c1 (B) 2 (1  c0 )
circle, the value of the contour integral (C) 2 jc1 (D) 2 j (1  c0 )
1
C Re ( z ) dz is _______.
2i 7.81 The residue of the function

7.76 The real part of an analytic function 1


f ( z) 
f ( z ), where z  u  iv is given by ( z  2) ( z  2) 2
2

e y cos ( x). The imaginary part of f ( z ) at z  2 is


is 1 1
(A) (B)
(A) e y cos ( x) (B) e y sin ( x) 32 16
(C)  e y sin ( x) (D)  e y sin ( x) 1 1
(C) (D)
7.77 C is a closed path in the z-plane given 16 32
by z  3 . The value of the integral 7.82 If the semi-circular contour D of radius
 z2  z  j4  2 is as shown in the figure, then the
C  z  j 2  dz is 1
value of the integral  ( s 2
 1)
ds is
(A)  4 (1  j 2) (B) 4 (3  j 2) D

(C)  4 (3  j 2) (D) 4 (1  j 2)
7.78 The value of the integral
 3z  4
C ( z 2  4 z  5) dz,
where C is the circle z  1 is given by
1
(A) 0 (B)
10 (A) j (B)  j 
4
(C) (D) 1 (C)   (D) 
5
 
Complex Variables | 11 
7.83 The equation sin ( z )  10 has

(A) no real (or) complex solution.


(B) exactly two distinct complex
solution.
(C) a unique solution.
(D) an infinite number of complex
solutions.
7.84 The value of the contour integral
1

z  j 2
z 4
2
dz in positive sense is

j 
(A) (B)
2 2
 j 
(C) (D)
2 2
7.85 For the function of a complex variable
w  ln z (where, w  u  jv, z  x  jy ),
the u  constant lines get mapped in the
z-plane as
(A) set of radial straight lines.
(B) set of concentric circles.
(C) set of confocal hyperbolas.
(D) set of confocal ellipses.

 
12 | Engineering Mathematics  
Answer Key : Complex Variables
7.1 A 7.2 D 7.3 D 7.4 B 7.5 B
7.6 A 7.7 D 7.8 B 7.9 B 7.10 A
7.11 A 7.12 D 7.13 – 0.33 7.14 – 0.25 7.15 B
7.16 D 7.17 C 7.18 C 7.19 A 7.20 D
7.21 B 7.22 1 7.23 B 7.24 C 7.25 C
7.26 D 7.27 A 7.28 B 7.29 D 7.30 B
7.31 B 7.32 C 7.33 A 7.34 C 7.35 A
7.36 D 7.37 C 7.38 A 7.39 A 7.40 A
7.41 B 7.42 B 7.43 A 7.44 B 7.45 B
7.46 –1 7.47 A 7.48 A 7.49 B 7.50 C
7.51 C 7.52 B 7.53 C 7.54 A 7.55 B
7.56 A 7.57 C 7.58 B 7.59 B 7.60 B
7.61 B 7.62 C 7.63 C 7.64 B 7.65 D
7.66 D 7.67 D 7.68 B 7.69 B 7.70 D
7.71 B 7.72 1 7.73 0 7.74 – 133.87 7.75 0.5
7.76 B 7.77 C 7.78 A 7.79 C 7.80 D
7.81 A 7.82 A 7.83 B 7.84 D 7.85 B



 
 
 
 

8.1 Match the following and choose the 8.3 A numerical solution of the equation
correct combination. f ( x )  x  x  3  0 can be obtained
Group-I using Newton-Raphson’s method. If the
E. Newton-Raphson’s method starting value is x  2 for the iteration,
F. Runge-Kutta method the value of x that is to be used in the
G. Simpson’s rule next step is
H. Gauss elimination (A) 0.306 (B) 0.739
Group-II (C) 1.694 (D) 2.306
8.4 Consider a differential equation
1. Solving non-linear equation
dy ( x )
2. Solving linear simultaneous  y ( x)  x
dx
equations
with the initial condition y(0)  0.
3. Solving ordinary differential
Using Euler’s first order method with a
equations
step size of 0.1, the value of y(0.3) is
4. Numerical integration method
(A) 0.01 (B) 0.031
5. Interpolation
(C) 0.0631 (D) 0.1
6. Calculation of Eigen values
8.5 The recursion relation to solve x  e x
(A) E-6, F-1, G-5, H-3
using Newton-Raphson’s method is
(B) E-1, F-6, G-4, H-3
(A) xn 1  e  xn
(C) E-1, F-3, G-4, H-2
(D) E-5, F-3, G-4, H-1 (B) xn 1  xn  e  xn
8.2 The equation x3  x2  4 x  4  0 is to e xn
(C) xn1  (1  xn )
be solved using the Newton-Raphson’s 1  e xn
method. If x  2 is taken as the initial x 2  e  xn (1  xn )  1
(D) xn 1  n
approximation of the solution, then the xn  e  xn
next approximation using this method 8.6 The Newton-Raphosn method is used to
will be solve the equation
2 4 f ( x)  x 3  5 x 2  6 x  8  0 .
(A) (B)
3 3 Taking the initial guess as x  5, the
3
(C) 1 (D) solution obtained at the end of the first
2 iteration is _______.

 
2 | Engineering Mathematics  
8.7 The ordinary differential equation 8.11 The function f ( x)  e x  1 is to be
dx solved using Newton-Raphson’s
  3 x  2, with x(0)  1
dt method. If the initial value of x0 is taken
is to be solved using the forward Euler 1.0, then the absolute error observed at
method. The largest time step that can 2nd iteration is _______.
be used to solve the equation without
8.12 The order of error in Simpson’s rule for
making the numerical unstable is
numerical integration with a step size h
_______.
is
8.8 Equation e x  1  0 is required to be
(A) h (B) h2
solved using Newton-Raphson’s method
with an initial guess x0  1 . Then, after (C) h3 (D) h4

one step of Newton-Raphson’s method, 8.13 We wish to solve x 2  2  0 be Newton-


estimate x1 of the solution will be given Raphson technique. Let the initial guess
by be x0  1.0. Subsequent estimated x1
(A) 0.71828 (B) 0.36784 will be
(C) 0.20587 (D) 0.00000 (A) 1.414 (B) 1.5

8.9 Let x 2  117  0. The iterative steps for (C) 2.0 (D) 3.0
the solution using Newton-Raphson’s 8.14 The value of the function f ( x) are
method is given by tabulated below
1 117  x 0 1 2 3
(A) xk 1   xk  
2 xk  f ( x) 1 2 1 10
117 Using Newton’s forward difference
(B) xk 1  xk 
xk formula, the cubic polynomial that can
xk be fitted to the above data is
(C) xk 1  xk 
117 (A) 2 x3  7 x 2  6 x  2
1 117  (B) 2 x3  7 x 2  6 x  2
(D) xk 1  xk   xk  
2 xk 
(C) x3  7 x 2  6 x 2
8.10 When the Newton-Raphson’s method is
(D) 2 x3  7 x 2  6 x  1
applied to solve the equation
8.15 Starting from x0  1, one step of
f ( x)  x3  2 x  1  0,
Newton-Raphson’s method in solving
The solution at the end of the iteration
the equation x3  3x  7  0 gives the
with the initial guess values as x0  1.2
next value ( x1 ) as
is
(A) – 0.82 (B) 0.49 (A) x1  0.5 (B) x1  1.406

(C) 0.705 (D) 1.69 (C) x1  1.5 (D) x1  2

 
Numerical Methods | 3 
8.16 A calculator has accuracy up to 8 digits 8.23 Consider an ordinary differential
after decimal place. The value of dx
2
equation  4t  4. If x  x0 at t  0,
dt
0
sin x dx when evaluated using this
the increment in x is calculated using
calculator by Trapezoidal method with 8 Runge-Kutta fourth order multistep
equal intervals to 5 significant digits is method with a step size of h  0.2 is
(A) 0.00000 (B) 1.0000 (A) 0.22 (B) 0.44
(C) 0.00500 (D) 0.00025 (C) 0.66 (D) 0.88
31
8.17 The integral  dx, when evaluated by 8.24 Simpson’s
1
rule is used to integrate
1 x
3
1 3 9
using Simpson’s rule on two equal the function f ( x )  x 2  between
3 5 5
sub-intervals each of length 1, equals x  0 and x  1 using the least number
(A) 1.000 (B) 1.098 of equal sub-intervals. The value of the
(C) 1.111 (D) 1.120 integral is _______.
8.18 Match the CORRECT pairs 8.25 Using a unit step size, the value of
Numerical integration Order of fitting 2

scheme polynomial
integral 
1
x ln x dx by Trapezoidal rule
P. 3 1. First is _______.
Simpson’s rule
8 8.26 Newton-Raphson method is used to find
Q. Trapezoidal rule 2. Second the roots of the equations,
R. 1 3. Third x3  2 x2  3x  1  0.
Simpson’s rule
3 If the initial guess is x0  1 then the
(A) P-2, Q-1, R-3 (B) P-3, Q-2, R-1 value of x after 2nd iteration is _______.
(C) P-1, Q-2, R-3 (D) P-3, Q-1, R-2 8.27 Solve the equation x  10cos ( x) using
8.19 Using the Trapezoidal rule and dividing the Newton-Raphson method. The initial
the interval of integration into three 
guess is x  . The value of the
equal sub-intervals, the define integral 4
1 predicted root after the first iteration, up
1
x dx is _______.
to second decimal, is _______.
4
8.20 The value of 2.5
ln ( x) dx calculated 8.28 Numerical integration using Trapezoidal
rule gives the best result for a single
using the Trapezoidal rule with five sub- variable function, which is
intervals is _______.
(A) Linear (B) Parabolic
31
8.21 The define integral  dx is evaluated (C) Logarithmic (D) Hyperbolic
1 x
8.29 The error in numerically computing the
using Trapezoidal rule with a step size n
of 1. The correct answer is _______. integral  0
(sin x  cos x) dx using the
8.22 The real root of the equation, Trapezoidal rule with three intervals of
5 x  2 cos x  1  0 (up to two decimal equal length between 0 and  is
accuracy) is _______. _______.

 
4 | Engineering Mathematics  
8.30 The root of the function f ( x)  x3  x  1 What would be the value of y at x  1?
obtained after first iteration on (A) 1.33 (B) 1.67
application of Newton-Raphson scheme (C) 2.00 (D) 2.33
using an initial guess of x0  1 is 8.34 The square root of a number N is to be
(A) 0.682 (B) 0.686 obtained by applying the Newton-
(C) 0.750 (D) 1.000 Raphson’s iteration to the equation
8.31 An explicit forward Euler method is x 2  N  0 . If i denotes the iteration
used to numerically integrate the index, the correct iterative method will
dy be
differential equation  y using time
dt 1 N
step of 0.1. With initial condition (A) xi 1   xi  
2 xi 
y(0)  1, the value of y (1) computed by
this method is _______ (correct to two 1 N
(B) xi 1   xi2  2 
decimal places). 2 xi 
8.32 Given a  0, we wish to calculate its
1 N2 
1 (C) xi 1   xi  
reciprocal value by using Newton- 2 xi 
a
Raphson method for f ( x)  0 . 1 N
(D) xi 1   xi  
(i) The Newton-Raphson’s algorithm 2 xi 
for the function will be dx 1.5

1 a
8.35 The estimate of
0.5 x 
obtained using
(A) xk 1   xk  
2 xk  Simpson’s rule with three-point function
evaluation exceeds the exact value by
a 2
(B) xk 1  xk  xk (A) 0.235 (B) 0.068
2
(C) xk 1  2 xk  axk2 (C) 0.024 (D) 0.012
8.36 The magnitude as the error (correct to
a
(D) xk 1  xk  xk2 two decimal places) in the estimation of
2 4
(ii) For a  7 and starting with x0  0.2, integral 
0
( x 4  10) dx using Simpson
the first two iterations will be 1/3rd rule is _______. (Take the step
(A) 0.11, 0.1299 (B) 0.12, 0.1392 length as 1)
(C) 0.12, 0.1416 (D) 0.13, 0.1428 8.37 Newton-Raphson method is to be used
dy to find root of equation
8.33 The differential equation  0.25 y 2 is
dx 3x  e x  sin x  0 .
to be solved using the backward If the initial trial value for the root is
(implicit) Euler’s method with the taken as 0.333, the next approximation
boundary condition y  1 at x  0 and for the root would be _______. (Note :
with a step size of 1. Answer up to three decimal)

 
Numerical Methods | 5 
du 8.43 Using the Trapezoidal rule and 4 equal
8.38 Consider the equation  3t 2  1 with
dt intervals (n  4), the calculated value of
u  0 at t  0 . This is numerically the integral (rounded to the first place of
solved by using the forward Euler 

method with a step size,  t  2 . The


decimal) 0
sin  d  is

absolute error in the solution at the end (A) 1.7 (B) 1.9
of the first time step is _______. (C) 2.0 (D) 2.1
8.39 The quadratic equation 2 x  3x  3  0 2
0.5

3

is to be solved numerically starting with 8.44 The value of the integral e x dx


0.1

an initial guess as x0  2 . The new evaluated by Simpson’s rule suing 4


estimate of x after the first iteration sub-intervals (up to 3 digits after the
using Newton-Raphson method is decimal point) is _______.
_______. 8.45 Consider the following differential
8.40 For step-size, x  0.4, the value of equation
1 dy
following integral using Simpson’s  x  ln ( y ); y  2 at x  0
3 dx
rule is _______. The solution of this equation at x  0.4
0.8
 (0.2  25 x  200 x  675 x  900 x  400 x ) dx using Euler method with a step size of
2 3 4 5
0
h  0.2 is _______.
8.41 In Newton-Raphson iterative method,
the initial guess values ( xini ) is 8.46 The solution of the non-linear equation
considered as zero while finding the x3  x  0
roots of the equation, is to be obtained using Newton-Raphson
f ( x)   2  6 x  4 x  0.5 x
2 3
method. If the initial guess is x  0.5,
The correction,  x to be added to xini the method converges to which one of
the following values
the first iteration is _______.
8.42 The iteration step in order to solve for (A) – 1 (B) 0
the cube roots of a given number ‘N’ (C) 1 (D) 2
using the Newton-Raphson’s method is 8.47 Values of f ( x) in the interval [0, 4] are
1 given below.
(A) xk 1  xk  ( N  xk3 )
3
x 0 1 2 3 4
1 N
(B) xk 1   2 xk  2  f ( x) 3 10 21 36 55
3 xk 
1 Using Simpson’s 1/3 rule with a step
(C) xk 1  xk  ( N  xk3 )
3 size of 1, the numerical approximation
(rounded off to the second decimal
1 N
(D) xk 1   2 xk  2  4
3 xk  place) of 
0
f ( x) is _______.

 
6 | Engineering Mathematics  
8.48 Which one of the following is an 8.50 The Newton-Raphson method is used to
iterative technique for solving a system find the roots of the equation
of simultaneous linear algebraic f ( x)  x  cos x; 0  x  1
equations? If the initial guess for the root is 0.5,
(A) Gauss elimination then the value of x after the first
(B) Gauss-Jordan iteration is
(C) Gauss-Seidel (A) 1.02 (B) 0.62
(D) LU decomposition (C) 0.55 (D) 0.38
8.52 For solving algebraic and transcendental
8.49 Match the application to appropriate
equation which one of the following is
numerical method.
used?
Applications Numerical Method
(A) Coulomb’s theorem
P1 Numerical M1 Newton- (B) Newton-Raphson method
integration Raphson’s method
(C) Euler’s method
P2 Solution to a M2 Runge-Kutta (D) Stokes’s theorem
transcendental method
equation
P3 Solution to a M3 1
Simpson’s rule
system of 3
linear
equations
P4 Solution to a M4 Gauss Elimination
differential method
equation

(A) P1-M3, P2-M2, P3-M4, P4-M1


(B) P1-M3, P2-M1, P3-M4, P4-M2
(C) P1-M4, P2-M1, P3-M3, P4-M4
(D) P1-M2, P2-M1, P3-M3, P4-M4
8.50 A root of the equation x 4  3x  1  0
needs to be found using the Newton-
Raphson method. If the initial guess x0,
is taken as 0, then the new estimate, x1
after the first iteration is
1 1
(A) (B) 
3 3
(C) 3 (D)  3

 
Numerical Methods | 7 

Answer Key : Numerical Methods


8.1 C 8.2 B 8.3 C 8.4 B 8.5 C
8.6 4.2903 8.7 0.66 8.8 A 8.9 A 8.10 C
8.11 1.657 8.12 D 8.13 B 8.14 D 8.15 C
8.16 A 8.17 C 8.18 D 8.19 1.1 8.20 1.7532
8.21 1.16 8.22 0.54 8.23 D 8.24 2 8.25 0.54
8.26 0.3043 8.27 1.56 8.28 A 8.29 0.186 8.30 C
(i) C,
8.31 2.59 8.32 8.33 C 8.34 A 8.35 D
(ii) B
8.36 0.53 8.37 0.36 8.38 8 8.39 1 8.40 1.367
8.41 0.33 8.42 B 8.43 B 8.44 0.384 8.45 2.33
8.46 A 8.47 94.67 8.48 C 8.49 B 8.50 A
8.51 D 8.52 B



You might also like